09.01.2014 Aufrufe

7d_jun_Arbeitsheft 3_Loesungen.pdf - Helmholtz Gymnasium Bonn

7d_jun_Arbeitsheft 3_Loesungen.pdf - Helmholtz Gymnasium Bonn

7d_jun_Arbeitsheft 3_Loesungen.pdf - Helmholtz Gymnasium Bonn

MEHR ANZEIGEN
WENIGER ANZEIGEN

Erfolgreiche ePaper selbst erstellen

Machen Sie aus Ihren PDF Publikationen ein blätterbares Flipbook mit unserer einzigartigen Google optimierten e-Paper Software.

Zuordnungen<br />

Graphen lesen und darstellen 1<br />

1<br />

Fünf zylindrische Gefäße stehen auf einer Treppe und werden gleichzeitig und gleichmäßig mit<br />

Wasser gefüllt. Welcher Füllhöhengraph gehört zu welchem Gefäß? (Die Höhe wird immer von der<br />

Nulllinie aus gemessen.)<br />

4 2 3 5 1<br />

Höhe<br />

1 2<br />

3<br />

4<br />

5<br />

0<br />

a b c d e<br />

Zeit<br />

2<br />

a) Die Bilder (I) und (II) zeigen die Querschnitte<br />

von zwei Berghängen. Während Paul der<br />

v<br />

v<br />

Pistenschreck hinunterfährt, wird ein Zeit-<br />

Geschwindigkeits-Diagramm aufgezeichnet.<br />

Welches der Diagramme gehört zu Bahn (I)<br />

t<br />

(II)<br />

t<br />

bzw. (II)?<br />

v<br />

v<br />

(I)<br />

(II)<br />

t<br />

(I)<br />

t<br />

b) Welche Bahn gehört zu dem t-v-Diagramm?<br />

v<br />

(II)<br />

t (I) (II) (III)<br />

c) Zeichne ein t-v-Diagramm zu der gegebenen<br />

Bahn.<br />

v<br />

t<br />

d) Zeichne eine Bahn zu dem t-v-Diagramm.<br />

v<br />

t


Zuordnungen<br />

Graphen lesen und darstellen 2<br />

<br />

Martinus Zack wohnt in Abelsweiler und<br />

besucht nachmittags regelmäßig seine<br />

Kunden. Er zeichnet seine Fahrten genau auf.<br />

a)<br />

Entfernung von Abelsweiler (in km)<br />

80<br />

70<br />

60<br />

50<br />

40<br />

30<br />

20<br />

Cantorshausen<br />

Abelsweiler 14 km<br />

21 km<br />

Besselberg<br />

32 km<br />

Eulerwald<br />

Gaussdorf<br />

13 km<br />

10 Uhrzeit<br />

14.00 15.00 16.00 17.00 18.00<br />

Wie weit ist Cantorshausen bzw. Eulerwald von Abelsweiler entfernt?<br />

Welches ist der zweite Ort, den Herr Zack besucht?<br />

Wie lange hält er sich in Gaussdorf, wie lange in Eulerwald auf?<br />

Wie lange fährt er von Gaussdorf nach Besselberg?<br />

35 km; 67 km<br />

Besselberg<br />

20 min; 15 min<br />

40 min<br />

b) Welcher der Graphen 1 oder 2 gehört zu welchem Fahrtprotokoll (I), (II) bzw. (III)?<br />

Ergänze den fehlenden Graphen im Diagramm.<br />

(I) A C G E B A<br />

an 14.30 15.40 16.10 17.10 18.00<br />

ab 14.00 15.00 16.00 16.30 17.40<br />

(II) A E G C B A<br />

an 15.00 15.30 16.40 17.05 18.00<br />

ab 14.00 15.15 16.10 16.55 17.35<br />

(III) A B E G C A<br />

an 14.20 15.15 15.50 16.50 18.00<br />

ab 14.00 14.35 15.40 16.10 17.30<br />

1 (III) j<br />

2 j(I)<br />

80<br />

70<br />

60<br />

50<br />

40<br />

30<br />

20<br />

Entfernung von Abelsweiler (in km)<br />

(II)<br />

1 2<br />

10 Uhrzeit<br />

14.00 15.00 16.00 17.00 18.00<br />

2


Zuordnungen<br />

Graph – Tabelle – Formel 1<br />

1<br />

Zeichne die Graphen zu den beiden Zuordnungen,<br />

die durch die Tabellen gegeben sind.<br />

a) x y b) x y<br />

0 0,5 0 4<br />

1 0,75 1 3,5<br />

2 1,5 2 3<br />

3 2,75 3 2,5<br />

4 4,5 4 2<br />

4<br />

3<br />

2<br />

1<br />

y<br />

b)<br />

a)<br />

x<br />

1 2 3 4<br />

2<br />

Vervollständige die Tabellen, die zu den beiden Graphen gehören.<br />

y<br />

a) x y b) x y<br />

− 4<br />

a)<br />

1 0,5 0 3,5<br />

− 3<br />

2 2 1 3,25<br />

− 2<br />

− 1<br />

b)<br />

x<br />

3 3,5 2 2,75<br />

4 5 3 1,75<br />

5 6,5 4 0,25<br />

1 2 3 4<br />

3<br />

Welche Graphen, Tabellen und Formeln gehören zusammen?<br />

a) y<br />

b) y<br />

(1) x y<br />

4<br />

4<br />

1 0,8<br />

3<br />

3<br />

2<br />

2<br />

2 1,6<br />

1<br />

1<br />

3 2,4<br />

x<br />

x<br />

4 3,2<br />

1<br />

2 3 4<br />

1<br />

2 3 4<br />

(2) x y<br />

1 3,75<br />

2 3<br />

3 1,75<br />

4 0<br />

(I) y = 4 – 0,25 x 2<br />

(II) y = 3 – 0,25 x<br />

(III) y = 0,5 x + 2<br />

(IV) y = 0,8 x<br />

c) y<br />

d) y<br />

(3) x y<br />

4<br />

3<br />

2<br />

1<br />

x<br />

1<br />

2 3 4<br />

4<br />

3<br />

2<br />

1<br />

1 2,5<br />

2 3<br />

3 3,5<br />

x<br />

4 4<br />

1<br />

2 3 4<br />

(4) x y<br />

1 2,75<br />

2 2,5<br />

3 2,25<br />

4 2<br />

a)<br />

b)<br />

c)<br />

d)<br />

(3) (III)<br />

(4) (II)<br />

(2) (I)<br />

(1) (IV)<br />

4<br />

Fehlerteufel: Welche<br />

Tabellenwerte wurden<br />

falsch berechnet?<br />

Die zugehörigen<br />

Buchstaben ergeben<br />

das Lösungswort.<br />

y = 0,75 x + 1 y = 7 – 0,2 x 2 y = 0,5 x 2 + 2 x<br />

x y x y x y<br />

1 1,75 P 1 6,8 S 1 2,5 F<br />

2 5,2 F 2 6,2 T 2 6,2 E<br />

3 3,52 O 3 5,2 U 3 10,5 I<br />

4 4 T 4 3,6 R 4 15 L<br />

5 4,75 E 5 2,2 M 5 22,5 E<br />

Lösungswort: FORMEL


Zuordnungen<br />

Graph – Tabelle – Formel 2<br />

<br />

Die Strecken des Diagramms sind die Graphen von Zuordnungen. Ergänze die Tabellen.<br />

a) x 0 1 2 3 4 y<br />

y 0 1 2 3 4<br />

b) x 4 5 6 7 8<br />

y 4 4,25 4,5 4,75 5<br />

c) x 1 2 3 4 5<br />

y 0 0,5 1 1,5 2<br />

d) x 8 9 10 11 12<br />

y 5 4,75 4,5 4,25 4<br />

e) x 6 6,5 7 7,5 8<br />

y 0 – 1 – 2 – 3 – 4<br />

10<br />

9<br />

8<br />

7<br />

6<br />

5<br />

b<br />

d<br />

4<br />

3<br />

a<br />

2<br />

1<br />

c<br />

–1 1 2 3 4 5 6 7 8 9 10 11 12 13 14 15 16<br />

e<br />

–2<br />

–3<br />

–4<br />

Entscheide, welche der Formeln zu den Graphen bzw. Tabellen gehört.<br />

y = – 0,25 · x + 7 y = – 2 · x + 12 y = 0,25 · x + 3 y = x y = 0,5 · x – 0,5<br />

d e b a c<br />

x<br />

2<br />

Vervollständige zu jeder Zuordnung die Tabelle und zeichne die Graphen. Runde auf zwei Stellen<br />

nach dem Komma.<br />

a) y = 0,25 x<br />

y<br />

x 2 3 4 5 6<br />

y 0,5 0,75 1 1,25 1,5<br />

7<br />

e)<br />

b) y = 0,5 x + 4<br />

x 2 2,5 3 3,5 4<br />

y 5 5,25 5,5 5,75 6<br />

auf zwei Stellen<br />

nach dem c) y = – 0,25 x + 6,5<br />

Komma gerundet x 4 4,5 5 5,5 6<br />

wegen Aufgabenstellung<br />

und<br />

y 5,5 5,38 5,25 5,13 5<br />

Platzmangel<br />

d) y = 3 · x 2 – 24 x + 49,5<br />

x 3 3,5 4 4,5 5<br />

y 4,5 2,25 1,5 2,25 4,5<br />

e) y = 0,5 x · (8 – x)<br />

x 0 1 2 3 4 5 6 7 8<br />

y 0 3,5 6 7,5 8 7,5 6 3,5 0<br />

6<br />

5<br />

4<br />

3<br />

2<br />

1<br />

b)<br />

d)<br />

a)<br />

1 2 3 4 5 6 7 8<br />

c)<br />

x<br />

4


Zuordnungen<br />

Ausgleichsgeraden<br />

1<br />

Eine Messreihe wurde in einem t-v-Diagramm dargestellt. Es sollen weitere Werte geschätzt werden.<br />

Zeichne eine Ausgleichsgerade und ermittle mit ihr die fehlenden v-Werte.<br />

a) b) c)<br />

4<br />

v<br />

4<br />

v<br />

4<br />

v<br />

3<br />

3<br />

3<br />

2<br />

2<br />

2<br />

1<br />

t<br />

1<br />

t<br />

1<br />

t<br />

1 2 3 4<br />

1 2 3 4<br />

1 2 3 4<br />

t 1,5 2,5 3,5 t 1,5 2,5 3,5 t 1,5 2,5 3,5<br />

v 1,1 1,8 2,5 v 1,9 2,5 3,1 v 2,8 2,3 1,8<br />

2<br />

Bei Schülerübungen im Physikunterricht haben Max und Moritz die Ausdehnung von belasteten<br />

Schraubenfedern untersucht. Dabei stellten sie, jeder mit einer anderen Feder, zwei Messreihen auf,<br />

bei denen die belastende Kraft F und die Verlängerung s gemessen wurde.<br />

Trage die Messwerte in das Koordinatensystem ein, zeichne die Ausgleichsgeraden und beantworte<br />

dann die Fragen.<br />

Max<br />

F in N 2 3 5 6 7<br />

s in cm 1,4 1,9 2,8 3,5 4,2<br />

Moritz<br />

F in N 0,5 1,5 2 3 4<br />

s in cm 0,5 2,0 2,7 3,6 5,1<br />

7<br />

6<br />

s<br />

Moritz<br />

Welche Verlängerungen ergeben sich bei<br />

Max, wenn die Kraft 0,5 N, 1 N oder 4 N<br />

beträgt?<br />

0,3 cm; 0,6 cm; 2,4 cm<br />

5<br />

4<br />

3<br />

2<br />

1<br />

Max<br />

1 2 3 4 5 6 7<br />

F<br />

Welche Verlängerungen ergeben sich bei<br />

Moritz, wenn die Kraft 1 N, 2,5 N oder 5 N<br />

beträgt?<br />

1,25 cm; 3,1 cm; 6,25 cm<br />

Versuche eine Formel zu finden, mit der man<br />

die Verlängerung abhängig von der Kraft<br />

ausrechnen kann.<br />

Max: s = 0,6 · F<br />

Moritz: s = 1,25 · F


Zuordnungen<br />

Proportionale Zuordnungen<br />

<br />

Welche der Zuordnungen können proportional sein, welche nicht? Kreuze an. Zeichne die Graphen<br />

der proportionalen Zuordnungen.<br />

a)<br />

b)<br />

c)<br />

d)<br />

y<br />

x y x y x y x y<br />

4<br />

1 0,6 0,5 0,3 0,5 1,2 0,5 0,6<br />

2 1,4 1 0,6 1,5 1,6 1,5 1,8<br />

3<br />

3 2,4 2,5 1,5 2 1,8 2,5 3,0<br />

4 3,6 3,5 2,1 4 2,6 3,5 4,2<br />

2<br />

d)<br />

b)<br />

j ja<br />

j nein<br />

j ja<br />

j nein<br />

j ja<br />

j nein<br />

j ja<br />

j nein<br />

1<br />

x<br />

1 2 3 4<br />

2<br />

Ergänze die Tabellen so, dass die Zuordnungen proportional sind.<br />

x 1,5 2,7 3,6 4,8 5,7 6,0 x 7 11 23 31 36 40<br />

y 0,5 0,9 1,2 1,6 1,9 2 y 16,1 25,3 52,9 71,3 82,8 92<br />

<br />

Welche Graphen, Tabellen und Rechenvorschriften gehören zusammen?<br />

4<br />

3<br />

2<br />

1<br />

y<br />

a)<br />

b)<br />

c)<br />

d)<br />

x<br />

1 2 3 4<br />

(1) x y<br />

0,5 0,3<br />

0,9 0,54<br />

2,0 1,2<br />

3,5 2,1<br />

(3) x y<br />

0,2 0,26<br />

1,2 1,56<br />

2,0 2,60<br />

3,3 4,29<br />

(2) x y<br />

0,4 0,44<br />

1,8 1,98<br />

3,0 3,30<br />

3,6 3,96<br />

(4) x y<br />

1,1 0,88<br />

2,4 1,92<br />

3,6 2,88<br />

4,0 3,20<br />

(I) y = 0,8 x<br />

(II) y = 1,3 x<br />

(III) y = 0,6 x<br />

(IV) y = 1,1 x<br />

a)<br />

b)<br />

c)<br />

d)<br />

(3) (II)<br />

(2) (IV)<br />

(4) (I)<br />

(1) (III)<br />

4<br />

Auch auf dem Mond ist die Gewichtskraft zur Masse eines Körpers proportional. Ein Raumfahrer mit<br />

voller Ausrüstung (114 kg) erfährt dort die Gewichtskraft 184,7 N. Welche Kraft wirkt<br />

a) auf ein Auto (976 kg),<br />

b) auf einen Elefanten (5,2 t),<br />

c) auf ein Stück Butter (250 g)?<br />

a) m in kg G in N b) m in kg G in N c) m in kg G in N<br />

114 184,7 114 184,7 114 184,7<br />

1 1,6 1 1,6 1 1,6<br />

976 1581,3 5200 8424,9 0,250 0,41<br />

6


Zuordnungen<br />

Rechnen mit proportionalen Zuordnungen<br />

1<br />

Löse die Aufgaben. Nur manche sind sinnvoll zu lösen. Deren Ergebnisse sind unten ohne Einheiten<br />

angegeben.<br />

a) Wie viel kosten 450 g?<br />

b) 400 g kosten 5,20 €.<br />

Wie viel erhält man für 9,10 €?<br />

j 200 g kosten j 3,60 €.<br />

Preis 5,20 € 1 € 9,10 €<br />

j 50 g kosten j 0,90 €.<br />

Menge 400 g 76,9 g 700 g<br />

j 450 g kosten j 8,10 €.<br />

Silber hat ein Volumen Masse Volumen 18 700<br />

von 4 cm 3 und 42 g 4 cm 3<br />

92,4<br />

9<br />

wiegt 42 g. Welches<br />

1 g 0,095 cm<br />

Volumen hat eine kleine<br />

Statue aus Silber, 73,5 g 7 cm 3 8,10<br />

die 73,5 g wiegt?<br />

c) 3 Flaschen Wein: 16,86 €<br />

d) 5 Flaschen Sekt:<br />

8 Flaschen Wein: ? €<br />

37,15 €<br />

37,15 € für<br />

Wie viele Flaschen 5 Flaschen<br />

für 66,87 €?<br />

1 € für<br />

0,13 Flaschen<br />

Anzahl 3 1 8<br />

66,87 € für<br />

9 Flaschen<br />

Betrag 16,86 € 5,62 € 44,96 €<br />

e)<br />

In 2 Wochen<br />

Urlaub habe ich 3 Kilo<br />

f) 3 kg Äpfel kosten 5,67 €.<br />

zugenommen.<br />

1 kg kostet 1,89 €<br />

.<br />

5 kg Äpfel kosten 9,45 €<br />

.<br />

keine<br />

Proportionalität<br />

Wie viel hat Herr S. nach 5 Wochen zugenommen?<br />

g) Benzinverbrauch<br />

Wieviel kosten<br />

h) An der Tankstelle<br />

Strecke<br />

Betrag 71,61 €<br />

26,6 l 350 km<br />

60 Liter?<br />

Abgabe 46,50 Liter<br />

1 l 13,16 km<br />

Abgabe 46,50 l 1 l 60 l<br />

49,4 l 650 km<br />

Betrag 71,61 € 1,54 € 92,40 €<br />

i) Zum Einsäen von 540 m 2 Rasenfläche<br />

braucht Herr Grün 12 kg Samen. Wie viel<br />

Samen braucht Herr Gras für seine 810 m 2 ?<br />

k) Oberstudienrat Eifrig<br />

hat mit 20 Schülern<br />

das Kapitel „Zuordnungen“<br />

in 5 Wochen keine<br />

Fläche 540 m 2 1 m 2 810 m 2 durchgenommen. Wie<br />

Proportionalität<br />

lange hätte er mit 25<br />

Samen 12 kg 0,02 kg 18 kg<br />

Schülern gebraucht?<br />

l) Ein kleiner Quader aus<br />

650<br />

44,96<br />

9,45<br />

7


Zuordnungen<br />

Antiproportionale Zuordnungen<br />

<br />

Welche der Zuordnungen können antiproportional sein, welche nicht? Kreuze an. Zeichne die<br />

Graphen der antiproportionalen Zuordnungen.<br />

y<br />

a) b) c) d)<br />

4<br />

x y x y x y x y<br />

0,5<br />

1,5<br />

3,63<br />

2,88<br />

0,5<br />

1<br />

3,6<br />

1,8<br />

1<br />

2<br />

4<br />

2<br />

0,5<br />

1<br />

4<br />

3<br />

3<br />

3 1,75 2 0,9 3 1, __ 3 2 1<br />

4 1 4 0,45 4 1 4 0,5<br />

2<br />

c) y = x<br />

4_<br />

j ja<br />

j nein<br />

j ja<br />

j nein<br />

j ja<br />

j nein<br />

j ja<br />

j nein<br />

1<br />

b) y = ___ 1,8 x<br />

x<br />

1 2 3 4<br />

2<br />

Ergänze die Tabellen so, dass die Zuordnungen antiproportional sind.<br />

x 2 4 5 6 9 10 x 2,4 3,0 6,4 8,0 15 24<br />

y 12 6 4,8 4 2, __ 6 2,4 y 40 32 15 12 6,4 4<br />

<br />

Welche Graphen, Tabellen und Rechenvorschriften gehören zusammen?<br />

y<br />

(1) x y (2) x y<br />

a)<br />

4 b)<br />

1 6 0,8 4<br />

2 3 1,6 2<br />

3<br />

3 2 3,2 1<br />

c)<br />

4 1,5 4,0 0,8<br />

2<br />

(3) x y (4) x y<br />

1<br />

0,4 5 0,5 3<br />

d)<br />

1,2 1,66 1,2 1,25<br />

x<br />

2,5 0,8 2 0,75<br />

1 2 3 4<br />

3,6 0,56 3,5 0,43<br />

(I) y = 2__<br />

x<br />

(II) y = 6__<br />

x<br />

(III) y = ___ 1,5<br />

x<br />

(IV) y = ___ 3,2<br />

x<br />

a) (4) (III)<br />

b)<br />

c)<br />

d)<br />

(3) (I)<br />

(2) (IV)<br />

(1) (II)<br />

4<br />

Bäcker Frischbrot stellt 300 Brezeln zu 40 g her. Wie viele Brezeln hätte er aus der gleichen<br />

Teigmenge herstellen können, wenn er sie<br />

a) 2 g leichter<br />

b) 2 g schwerer<br />

c) 5 g schwerer gemacht hätte?<br />

a) Gewicht Anzahl b) Gewicht Anzahl c) Gewicht Anzahl<br />

40 g 300 40 g 300 40 g 300<br />

1 g 12 000 1 g 12 000 1 g 12 000<br />

38 g 315 42 g 285 45 g 266<br />

8


Zuordnungen<br />

Rechnen mit antiproportionalen Zuordnungen.<br />

1<br />

Löse die Aufgaben. Nur manche sind sinnvoll zu lösen. Deren Ergebnisse sind unten ohne Einheiten<br />

angegeben.<br />

a)<br />

Unser Hafer b)<br />

Unser Hafer<br />

reicht für 189 Tage.<br />

reicht für 250<br />

Tage.<br />

Anzahl Zeit<br />

3 189 d<br />

1 567 d<br />

7 81 d<br />

Wie viele Pferde sind im Stall, wenn der<br />

Vorrat für 100 Tage reicht?<br />

Wie<br />

lange reicht<br />

das für uns?<br />

Vorrat 250 d 1 d 100 d<br />

Pferde 2 500 5<br />

c) Mit 32 Feuerwehrmännern<br />

wurde ein Brand in 6 Stunden<br />

gelöscht. Wie lange dauert das<br />

Löschen, wenn 48 Männer<br />

zum Einsatz kommen?<br />

keine Proportionalität<br />

d) Drei Planierraupen<br />

benötigen 21 Std.<br />

Eine Planierraupe<br />

benötigt 63 Std.<br />

Neun Planierraupen<br />

benötigen 7 Std.<br />

e) 54 Personen müssen<br />

für die Busfahrt je<br />

25 € bezahlen. Wie<br />

viel muss jeder zahlen,<br />

wenn nur 50<br />

Personen mitfahren?<br />

54 für 25 €<br />

1 für 1350 €<br />

50 für 27 €<br />

f) Zwei Bauarbeiter heben<br />

eine Grube mit den<br />

Maßen 2 m mal 2 m, die<br />

3 m tief ist, in 3 Stunden<br />

aus. Wie lange würden<br />

10 Arbeiter benötigen?<br />

nicht durchführbar<br />

g) Der Brenner einer Ölheizung verbraucht je<br />

Betriebsstunde 3,6 l Öl. eine Tankfüllung<br />

reicht für 1500 Betriebsstunden. Wie viele<br />

Stunden reicht die Tankfüllung, wenn ein<br />

neuer Brenner eingesetzt wird, der 3,2 l verbraucht?<br />

Öl 3,6 l 1 l 3,2 l<br />

Std. 1500 5400 1687,5<br />

h) Der Brenner einer Ölheizung verbraucht je<br />

Betriebsstunde 3,2 l Öl. Eine Tankfüllung<br />

reicht für 1500 Betriebsstunden. Welchen<br />

Verbrauch hat ein neuer Brenner je Stunde,<br />

wenn die Tankfüllung für 1600 Stunden<br />

reicht?<br />

Std. 1500 1 1600<br />

Öl 3,2 l 4800 l 3 l<br />

i) Ute hat für die Urlaubsreise<br />

Taschengeld gespart. Wenn sie<br />

täglich 9 € ausgibt reicht das Geld<br />

16 Tage. Wie lange reicht es bei<br />

6 € täglich?<br />

9 € für 16 d<br />

1 € für 144 d<br />

6 € für 24 d<br />

k)<br />

Mit 5 Leuten<br />

dauert unsere Wanderung<br />

3,5 Stunden.<br />

keine Proportionalität<br />

Wie lange dauert sie mit 7 Personen?<br />

l) In 14 Tagen können<br />

8 Gärtner einen Park<br />

anlegen. Wie viele<br />

Tage werden benötigt,<br />

wenn nur 7 Gärtner zur<br />

Verfügung stehen?<br />

Gärtner Tage<br />

8 14<br />

1 112<br />

7 16<br />

81<br />

24<br />

5<br />

1687,5<br />

16<br />

7<br />

3<br />

27


Zuordnungen<br />

Proportionale und antiproportionale Zuordnungen<br />

<br />

Welche der folgenden Tabellen bzw. Graphen gehören zu einer besonderen Zuordnung? Kreuze an.<br />

a) b) c) d) e)<br />

f) g) h) i) k)<br />

x y x y x y x y x y<br />

2 12 3 2 2 7 3 1 4 10<br />

3 8 5 4 3 5,5 6 6 6 15<br />

6 4 7 6 4 4 9 3 8 20<br />

10 2,4 9 8 5 2,5 12 9 10 25<br />

Zuordnung a) b) c) d) e) f) g) h) i) k)<br />

je mehr – desto mehr<br />

je mehr – desto weniger<br />

proportional<br />

antiproportional<br />

nichts davon<br />

2<br />

Welche der Zuordnungen ist proportional, welche antiproportional, welche weder proportional<br />

noch antiproportional?<br />

a) Anzahl 14 23 35 43 b) Gewicht in kg 8 13 17 20<br />

Preis in € 5,88 9,66 14,70 18,06 Preis in € 13,44 21,48 25,68 33,60<br />

proportional<br />

weder noch<br />

c) Anzahl 16 25 32 50 d) Stückzahl 15 25 35 45<br />

Dauer in h 7,5 4,8 3,75 2,4 Preis in € 1,18 1,40 1,54 1,65<br />

antiproportional<br />

weder noch<br />

<br />

Fülle die Tabellen aus. Gib auch Proportionalitätsfaktor bzw. Produkt an.<br />

a) Die Zuordnung ist proportional b) Die Zuordnung ist antiproportional<br />

x 7 10 12 15 x 4 6 8 16<br />

y 8,4 12 14,4 18 y 2,4 1,6 1,2 0,6<br />

Proportionalitätsfaktor: 1,2 Produkt: 9,6<br />

c) Die Zuordnung ist proportional. d) Die Zuordnung ist antiproportional.<br />

x 12 26 40 54 x 12 26 40 54<br />

y 9 19,5 30 40,5 y 42,25 19,5 12,675 9,389<br />

Proportionalitätsfaktor: 0,75 Produkt: 507<br />

0


Zuordnungen<br />

Rechnen mit Zuordnungen 1<br />

Gib bei folgenden Aufgaben immer zuerst die Zuordnung an. Überlege anschließend, ob sie proportional<br />

oder antiproportional ist.<br />

1<br />

Ein Traktor fährt eine bestimmte Wegstrecke.<br />

Dabei drehen sich die großen Hinterräder, die<br />

einen Umfang von 5,10 m haben, 1938-mal.<br />

Zuordnung: Umfang ° Drehungen<br />

Proportional oder antiproportional: antiproportional<br />

<br />

Wie oft drehen sich dabei die Vorderräder, deren Umfang 1,90 m<br />

beträgt?<br />

Umfang Drehungen<br />

5,10 m 1938<br />

1 m 9883,8<br />

1,90 m 5202<br />

2<br />

3<br />

„Sonderangebot! Hackfleisch, 500 g nur 1,75 €“, steht an der<br />

Metzgereitheke.<br />

Zuordnung: Gewicht °<br />

Preis<br />

Proportional oder antiproportional: proportional<br />

<br />

a) Frau Sparpfennig kauft 1300 g. Wie viel muss sie bezahlen?<br />

b) Frau Markus will 800 g vom Sonderangebot kaufen. „Darf<br />

es etwas mehr sein?” , fragt die Verkäufern. Anschließend<br />

bezahlt Frau Markus 3,01 €. Wie viel Gramm waren es mehr?<br />

„Warum regen sich alle Leute immer über die<br />

Benzinpreise auf?“, wundert sich Herr Witzigmann.<br />

„Ich tanke immer für 30 €.“<br />

Zuordnung: Preis ° Liter<br />

Proportional oder antiproportional: proportional<br />

<br />

Wieviel Liter Benzin erhält Herr Witzigmann für seine 30 € wenn<br />

a) 40 l Benzin 58,40 € kosten,<br />

b) 45 l Benzin 71,55 € kosten?<br />

a) Gewicht Preis<br />

500 g 1,75 €<br />

100 g 0,35 €<br />

1300 g 4,55 €<br />

b) Preis Gewicht<br />

1,75 € 500 g<br />

1 € 285,7 g<br />

3,01 € 860 g<br />

Es sind 60 g mehr<br />

a) Preis Liter<br />

58,40 € 40<br />

1 € 0,685<br />

30 € 20,548<br />

b) Preis Liter<br />

71,55 € 45<br />

1 € 0,629<br />

30 € 18,868<br />

4<br />

Um Papier zu sparen, beschließt Verleger Schragel das neueste<br />

Buch von Professor Dreistein „Mathe macht glücklich“ etwas<br />

enger zu drucken. So sollen 38 Zeilen pro Seite statt der üblichen<br />

36 Zeilen pro Seite gesetzt werden. Bei einem 36-Zeilen-Satz<br />

hätte das Buch 684 Seiten.<br />

Zuordnung: Zellen ° Seiten<br />

Proportional oder antiproportional: antiproportional<br />

<br />

a) Wie viele Seiten hat das Buch, wenn 38 Zeilen gesetzt werden?<br />

b) Wie viele Zeilen müssten pro Seite gesetzt werden, wenn das<br />

Buch nur 616 Seiten haben soll?<br />

a) Zeilen Seiten<br />

36 684<br />

1 24 624<br />

38 648<br />

b) Seiten Zeilen<br />

684 36<br />

1 24 624<br />

616 40<br />

Lösungen (ohne Einheiten und gerundet): 4,55; 18,9; 20,5; 40; 60; 648; 5202<br />

11


Zuordnungen<br />

<br />

Rechnen mit Zuordnungen 2<br />

Herr Frühlich hat Wein gekauft. Für 17 Flaschen<br />

„Matheberger Sorgenbrecher“ hat er 115,60 €<br />

bezahlt.<br />

a) Wie viel hätten 25 Flaschen von dieser Sorte<br />

gekostet?<br />

b) Wie viele Flaschen erhält man für 149,60 €?<br />

a) b)<br />

Flaschen Preis Preis Flaschen<br />

17 115,60 115,60 17<br />

1 6,80 1 0,15<br />

25 170 149,60 22<br />

2<br />

<br />

„Wenn wir in einer Woche 5 kg Kartoffeln essen,<br />

reicht unser Vorrat für 28 Wochen”, erklärt Oma<br />

Fanny.<br />

a) Wie lange reicht der Vorrat, wenn wöchentlich<br />

7 kg gegessen werden?<br />

b) Wie viel wird in der Woche gegessen, wenn<br />

der Vorrat 35 Wochen reicht?<br />

Im Mehrfamilienhaus „Teures Wohnen” wird die<br />

Miete nach Quadratmetern berechnet. Hinzu<br />

kommen für jede Wohnung Nebenkosten in<br />

Höhe von 95 €. Familie Maier bezahlt für ihre<br />

130 m 2 große Wohnung insgesamt 1057 €.<br />

a) Wie hoch ist die Gesamtmiete für die<br />

Nachbarwohnung (120 m 2 )?<br />

b) Nach einer Mieterhörung hat Herr Maier<br />

insgesamt 1105,10 € zu bezahlen, wobei die<br />

Nebenkosten gleich geblieben sind. Wie<br />

viele Quadratmeter hat die Wohnung von<br />

Familie Müller, die nach der Erhöhung insgesamt<br />

1182,80 € bezahlen muss?<br />

a) b)<br />

Menge Wochen Wochen Menge<br />

5 kg 28 28 5 kg<br />

1 kg 140 1 140 kg<br />

7 kg 20 35 4 kg<br />

a) 1057 € – 95 = 962 €<br />

m 2 Miete<br />

130 962 €<br />

1 7,40 €<br />

120 888 €<br />

8 8 8 €<br />

+ 9 5 €<br />

9 8 3 €<br />

b) 1105,10 € – 95 € = 1010,10 €<br />

1182,80 € – 95 € = 1087,80 €<br />

Miete m 2<br />

1010,10€ 130<br />

1 € 0,129<br />

1087,80 € 140<br />

4<br />

Eine Parkanlage kann von 10 Gärtnern in 12<br />

Tagen angelegt werden.<br />

a) Wie viele Tage werden benötigt, wenn nur 4<br />

Gärtner zur Verfügung stehen?<br />

b) Die Anlage ist schon nach 8 Tagen fertiggestellt.<br />

Wie viele Gärtner waren im Einsatz?<br />

c) Wie viele Tage werden insgesamt gebraucht,<br />

wenn von den 10 Gärtnern nach 4 Tagen 2<br />

wegen Krankheit ausfallen?<br />

d) Wie viele Tage werden insgesamt gebraucht,<br />

wenn den 10 Gärtnern, nach 3 Tagen 5<br />

Gärtner zu Hilfe kommen?<br />

a) b)<br />

Gärtner Tage Tage Gärtner<br />

10 12 12 10<br />

1 120 1 120<br />

4 30 8 15<br />

c) d)<br />

Gärtner Tage Gärtner Tage<br />

10 12 10 12<br />

10 8 10 9<br />

1 80 1 90<br />

8 10 15 6<br />

14 Tage 9 Tage<br />

Tipp zu c) und d): Nach 4 Tagen benötigen 10 Gärtner 8 Tage, nach 3 Tagen benötigen 10 Gärtner<br />

9 Tage.<br />

Die Lösungen (ohne Einheiten) sind hier angegeben. Die zugehörigen Buchstaben ergeben in der Reihenfolge der<br />

Aufgaben eine alte Bezeichnung für Dreisatzrechnung.<br />

140 22 4 30 20 9 983 170 14 15<br />

D E E E G I L R R T<br />

Lösungswort: REGELDETRI<br />

2


Zuordnungen<br />

Rechnen mit Zuordnungen 3<br />

1<br />

Herr Reichlich will seinen großen Swimmingpool<br />

leeren. Er benutzt dazu 3 Pumpen, die in 12<br />

Stunden 3600 l Wasser fördern. Wie viele Liter<br />

Wasser pumpen 8 Pumpen in 9 Stunden?<br />

Benutze die Tabelle zur Lösung.<br />

Überlege bei jedem Schritt, ob die zugrunde liegende<br />

Zuordnung proportional oder antiproportional<br />

ist.<br />

Pumpe Zeit Menge<br />

3 12 h 3600 l<br />

1 12 h 1200 l<br />

8 12 h 9600 l<br />

8 1 h 800 l<br />

8 9 h 7200 l<br />

2<br />

3 Maschinen fertigen 6 Bauteile in 4 Stunden.<br />

Wie lange dauert es, bis 10 Maschinen 15<br />

Bauteile produziert haben?<br />

zu sam<br />

men ge setz<br />

ter Drei satz<br />

Maschinen Teile Zeit<br />

3 6 4 h<br />

1 6 12 h<br />

10 6 1,2 h<br />

10 1 0,2 h<br />

10 15 3 h<br />

3<br />

Im letzten Jahr haben in der Verwaltung der Stadt Fröhlichberg 12 Büroangestellte in 15 Tagen 200<br />

Akten bearbeitet. Wie lange brauchen 9 Angestellte für 140 Akten?<br />

Angestellte Akten Tage<br />

12 200 15<br />

1 200 180<br />

9 200 20<br />

9 1 0,1<br />

9 140 14<br />

4<br />

Auf der Großbaustelle „Langer“ werden 14 Lkw benötigt, die 36 Tage lang jeden Tag 8 Fahrten<br />

machen.<br />

a) Wie viele Tage dauern die Arbeiten, wenn 2 Wagen ausfallen und nur 7 Fahrten pro Tag möglich<br />

sind?<br />

b) Die Arbeit soll in 32 Tagen fertig sein, wobei 9 Fahrten pro Tag möglich sind. Wie viele Lkw werden<br />

dann benötigt?<br />

c) Wie viele Tage dauern die Arbeiten, wenn nach 12 Tagen 2 Lkw ausfallen, wobei weiterhin 8<br />

Fahrten pro Tag durchgeführt werden?<br />

a) LKW Fahrten Tage b) Fahrten Tage LKW<br />

14 8 36 8 36 14<br />

1 8 504 8 1 504<br />

12 8 42 8 32 15,75<br />

12 1 336 1 32 126<br />

12 7 48 9 32 14<br />

c) LKW Tage<br />

14 36<br />

14 24<br />

1 336<br />

12 28<br />

28 + 12 = 40<br />

Lösungen (ohne Einheiten): 3; 14; 14; 40; 48; 7200<br />

13


Zuordnungen<br />

Terme 1<br />

<br />

Welche Tabellen, Terme und Wortbeschreibungen stellen dieselbe Zuordnung dar?<br />

a) x 1 2 3 4<br />

T(x) 2 6 12 20<br />

b) x 2 3 4 5<br />

T (x) 5 8 11 14<br />

c) x 1 2 3 4<br />

T (x) 8 10 12 14<br />

d) x 1 3 5 7<br />

T (x) 0 8 24 48<br />

(I) Jeder Zahl wird ihr Drei faches,<br />

vermindert um 1, zugeordnet.<br />

(II) Jeder Zahl wird ihr Pro dukt mit<br />

der um 1 größeren Zahl zugeordnet.<br />

(III) Jeder Zahl wird ihr Qua drat, vermindert<br />

um 1, zugeordnet.<br />

(IV) Jeder Zahl wird das Doppelte der<br />

um 3 größeren Zahl zugeordnet.<br />

(1) T (x) = x 2 – 1<br />

(2) T (x) = 2 (x + 3)<br />

(3) T (x) = x (x + 1)<br />

(4) T (x) = 3 x – 1<br />

a)<br />

b)<br />

(II)<br />

(I)<br />

(3)<br />

(4)<br />

c)<br />

d)<br />

(IV) (2)<br />

(III) (1)<br />

2<br />

Ergänze die Tabellen und finde einen Term. (Bei einer Aufgabe lässt sich kein Term finden.)<br />

a) n 1 2 3 4 5 6 b) n 1 2 3 4 5 6<br />

T (n) 1 3 5 7 9 11 T (n) 0 3 8 15 24 35<br />

2 n – 1<br />

n 2 – 1<br />

c) n 1 2 3 4 5 6 d) n 1 2 3 4 5 6<br />

T (n) 2 3 5 7 11 13 T (n) 1 3 7 15 31 63<br />

Primzahlen<br />

2 n – 1<br />

<br />

T (n) gibt die Anzahl der Kästchen an, aus denen die Muster bestehen. Zeichne das nächste Muster,<br />

finde einen Term und fülle die Tabelle aus.<br />

a) n 1 2 3 4 5 10<br />

T (n) 1 4 9 16 25 100<br />

Term: n 2<br />

b) n 1 2 3 4 5 10<br />

T (n) 3 6 11 18 27 102<br />

Term:<br />

n 2 + 2<br />

c) n 1 2 3 4 5 10<br />

T (n) 3 8 15 24 35 120<br />

Term:<br />

n 2 + 2 n<br />

4<br />

Max legt mit Streichhölzern Figuren. Wie viele hat er benutzt, wenn er bis zum 24. Quadrat gekommen<br />

ist?<br />

…<br />

1 2 3 4 5 6 24<br />

1 + 24 · 4 = 97<br />

4


Zuordnungen<br />

Terme 2<br />

1<br />

Gib einen Term für den Flächeninhalt und den Umfang der folgenden Figuren an.<br />

a)<br />

b)<br />

x<br />

4<br />

x 1<br />

x 1<br />

4 x<br />

1<br />

Flächeninhalt: 16 x<br />

Flächeninhalt: 10 x<br />

1<br />

x<br />

Umfang: 8 x + 8<br />

Umfang: 8 x + 8<br />

2<br />

Finde einen Term für den Flächeninhalt der nebenstehenden<br />

Figur.<br />

2,5 x 2<br />

x<br />

2 x<br />

x<br />

3<br />

Es soll ein Kantenmodell aus Draht gebaut werden. Bestimme je einen Term, der die Mindestlänge<br />

des Drahtes angibt, den man zum Bau der Modelle benötigt.<br />

a) Prisma<br />

b) Pyramide<br />

c) Pyramide auf Prisma<br />

3<br />

3<br />

3<br />

x 3 x<br />

3<br />

3<br />

x<br />

2<br />

x<br />

x<br />

2<br />

x<br />

x<br />

x<br />

x<br />

6 x + 6 3 x + 9 6 x + 15<br />

4<br />

Die Zeichnung stellt das Netz eines Quaders dar. Finde einen<br />

Term für die Oberfläche und das Volumen des zugehörigen<br />

Quaders.<br />

Oberfläche: 2 (15 x + 2 x 2 )<br />

Volumen:<br />

10 x 2 150 – 3 x 2<br />

<br />

x<br />

5<br />

2 x<br />

5<br />

Aus einem Quader sind Teile herausgefräst. Gib einen Term zur Berechnung des Volumens und der<br />

Oberfläche des Restkörpers an.<br />

a)<br />

b)<br />

10 3<br />

5<br />

x<br />

x 5<br />

x<br />

5<br />

x<br />

x<br />

6<br />

Volumen: Volumen: 150 – x 2<br />

Oberfläche: 190 + 12 x – 2 x 2 Oberfläche: 170<br />

Was fällt auf? Hängt nicht von x ab.<br />

15


Prozent- und Zinsrechnung<br />

Relativer Vergleich 1<br />

<br />

Färbe den jeweils angegebenen Teil der Fläche nach Augenmaß ein.<br />

a) 25 % b) 50 % c) 33 %<br />

2<br />

Gib den Prozentsatz der gefärbten Fläche an.<br />

a) b) c)<br />

j75 % j10 % j12,5 %<br />

<br />

Fülle die Tabelle aus.<br />

davon 50 % 10 % 90 % 25 % 33, __ 3 % 40 %<br />

330 € 165 € 33 € 297 € 82,50 € 110 € 132 €<br />

150 kg 75 kg 15 kg 135 kg 37,5 kg 50 kg 60 kg<br />

450 km 225 km 45 km 405 km 112,5 km 150 km 180 km<br />

90 ml 45 ml 9 ml 81 ml 22,5 ml 30 ml 36 ml<br />

510 cm 255 cm 51 cm 459 cm 127,5 cm 170 cm 204 cm<br />

1200 kg 600 kg 120 kg 1080 kg 300 kg 400 kg 480 kg<br />

4<br />

Gib die zugehörigen Anteile und Prozentsätze an.<br />

55 € von 550 € ___ 1<br />

j<br />

10 = 10 % a) 45 m von 300 m 3<br />

______<br />

20<br />

j<br />

b) 70 l von 210 l ______<br />

1<br />

jjj j<br />

j = % c) 400 m 2 von 600 m 2<br />

______<br />

2<br />

33, __ 3<br />

3<br />

3<br />

j<br />

d) 250 m von 2000 m ______<br />

1<br />

jjj j<br />

j = % e) 1000 € von 2500 €<br />

______<br />

2<br />

12,5<br />

8<br />

5<br />

j<br />

f) 20 m 2 von 400 m 2 ______<br />

1<br />

jjj<br />

j j<br />

= % g) 760 Stimmen von 800 Stimmen ______<br />

19<br />

5<br />

20<br />

20<br />

j<br />

h) 891 € von 900 € ______<br />

99<br />

______ j 11<br />

99<br />

100<br />

100<br />

j = jjj % i) 22 Stimmen von 200 Stimmen<br />

15<br />

j = jjj %<br />

66, __ 6<br />

j = jjj %<br />

40<br />

j = jjj %<br />

95<br />

j = jjj %<br />

11<br />

j = jjj %<br />

6


Prozent- und Zinsrechnung<br />

Relativer Vergleich 2<br />

1<br />

Ordne zu.<br />

a)<br />

​ 1__<br />

4 ​ 12,5 % b)<br />

0,017 1,7 %<br />

c)<br />

_______ ​<br />

1<br />

100 000 ​ 0,1 %<br />

1 25 % 0,17 3,7 % ​________<br />

1<br />

1000 000 ​ 10 ‰<br />

0,2 133,3 % _____ ​ 17 ​ 0,37 % 0,001 0,01 %<br />

1000<br />

​ 1__ ​ 100 % 0,037 1,7 % 0,0001 10 ppm<br />

8<br />

​ 4__ ​ 20 % 0,0037 17 % 0,01 1 ppm<br />

3<br />

2<br />

3<br />

Bestimme die Prozentsätze wie im Beispiel. Kürzen kann helfen.<br />

___ ​ 14<br />

35 ​= __ ​2 5 ​ = ​40 ____<br />

100 ​ = 40 % a) ____<br />

170 ​17 ​= ​j ______ 1<br />

10<br />

b) ____ ​ 18 ​= ​j ______<br />

6<br />

150<br />

50<br />

d) ____ ​ 16 ​= ​j<br />

320<br />

f) ____ ​ 10 ​= ​j<br />

125<br />

j ​ jjj = _______<br />

12<br />

​<br />

100<br />

______ 1<br />

j ​ jjj = _______ 5<br />

​<br />

20 100<br />

______<br />

2<br />

j ​ jjj = _______<br />

8<br />

​<br />

25 100<br />

12<br />

jjj ​ = jj % c) ___ ​ 63<br />

70<br />

5<br />

jjj ​ = jj % e) ​ 36<br />

8<br />

jjj ​ = jj % g) ___ ​ 16<br />

80<br />

j ​ jjj = _______ 10<br />

​<br />

100<br />

9<br />

______<br />

j ​ jjj 90<br />

= _______ ​<br />

10 100<br />

______<br />

1<br />

j ​ jjj = _______<br />

25<br />

​<br />

4 100<br />

______<br />

2<br />

j ​ jjj = _______<br />

20<br />

​<br />

10 100<br />

​= ​j<br />

____ ​= ​j<br />

144<br />

​= ​j<br />

jjj ​ = jj %<br />

jjj ​ = jj %<br />

jjj ​ = jj %<br />

jjj ​ = jj %<br />

Ein Fernseher kostet 1000 €. Der Preis wird zweimal hintereinander um 20 % erhöht. Wie viel kostet<br />

der Fernseher nach der zweiten Preiserhöhung. Mache zunächst einen Überschlag.<br />

Überschlag: jjjjj 1400 €<br />

10<br />

90<br />

25<br />

20<br />

20 % von 1000 € sind 200 €<br />

1200 € nach der ersten Erhöhung<br />

20 % von 1200 € sind 240 €<br />

1440 € nach der zweiten Erhöhung<br />

Preis nach der ersten Erhöhung: jjjjj 1200 € Preis nach der zweiten Erhöhung: jjjjj 1440 €<br />

4<br />

Für Überschlagsrechnungen im Kopf ist es häufig einfacher mit Anteilen zu rechen als mit<br />

Prozentangaben ​( 51 % ist ungefähr ​ 1__<br />

2 ​ )​. Ordne die Anteile den Prozentangaben zu.<br />

1,9 % 4,8 % 8,4 % 10,1 % 11 % 12,61 % 14,3 % 19,4 % 25,3 % 32 % 49,1 %<br />

​___<br />

1<br />

50 ​ ​1 ___<br />

20 ​ ___ ​1 12 ​ ___ ​1 10 ​ __ ​1 9 ​ __ ​1 8 ​ __ ​1 7 ​ __ ​1 5 ​ __ ​1 4 ​ __ ​1 3 ​ __ ​1 2 ​<br />

​___<br />

1<br />

50 ​ ___ ​1 20 ​ __ ​1 7 ​ ​1 __<br />

5 ​<br />

​__<br />

1 3 ​ __ ​1 8 ​<br />

​__<br />

1 2 ​ __ ​1 9 ​<br />

__ ​ 1 4 ​ ___ ​1 10 ​ ___ ​1 12 ​<br />

17


Prozent- und Zinsrechnung<br />

Prozentwert<br />

<br />

Berechne den Prozentwert in der Tabelle. Man muss nicht immer auf 1 % schließen.<br />

a) 24 % von 650 € b) 57 % von 1250 l c) 48 % von 3420 m<br />

Prozent Menge<br />

100 % 650 €<br />

4 % 26 €<br />

Prozent Menge<br />

Prozent Menge<br />

100 % 1250 l<br />

100 % 3420 m<br />

1 % 12,5 l<br />

4 % 136,8 m<br />

24 % 156 €<br />

57 % 712,5 l<br />

48 % 1641,6 m<br />

2<br />

Berechne den Prozentwert im Rechenschema.<br />

a) 3 % von 1577 € b) 16 % von 25 Mitgliedern c) 45 % von 10 Minuten<br />

15,77 €<br />

0,25<br />

0,1 min<br />

: 100 ∙ 3 : 100<br />

· 16 : 100<br />

· 45<br />

47,31 €<br />

1577 €<br />

3<br />

∙ ___<br />

100<br />

3 % von 1577 € sind jj<br />

25<br />

·____<br />

16<br />

100<br />

4<br />

16 % von 25 Mitgliedern<br />

sind 4 Mitglieder.<br />

10 min 4,5 min<br />

· ____ 45<br />

100<br />

45 % von 10 min sind<br />

4,5 min.<br />

<br />

Berechne den Prozentwert wie im Beispiel. Nutze gegebenenfalls den Taschenrechner.<br />

79 % von 450 € a) 67 % von 555 m<br />

· 0,79<br />

450 € ⎯⎯→<br />

jjj · 0,67<br />

335,50 €<br />

555 m ⎯⎯→ 371,85 €<br />

b) 11 % von 8000 kg c) 85 % von 25,20 €<br />

jjj · 0,11<br />

8000 kg ⎯⎯→ 8380 kg<br />

jjj · 0,85<br />

25,20 € ⎯⎯→ 21,42 €<br />

4<br />

Berechne. Wähle selbst einen Rechenweg, den du für sinnvoll hältst.<br />

a) 12 % von 20 000<br />

Wählerstimmen<br />

2400<br />

Wählerstimmen<br />

b) 85 % von 120 kg c) 0,3 % von 4000 ml<br />

102 kg 12 ml<br />

d) 190 % von 66,60 € e) 102 % von 35 m 2 f) 36 % von 450 ml<br />

126,54 € 35,7 m 2 162 ml<br />

8


Prozent- und Zinsrechnung<br />

Grundwert<br />

1<br />

Berechne den Grundwert in der Tabelle. Man muss nicht immer auf 1 % schließen.<br />

a) 45 % entpsrechen 90 € b) 64 % entsprechen 2 000 ml c) 125 % entsprechen 550 kg<br />

Prozent Menge<br />

45 % 90 €<br />

5 % 10 €<br />

Prozent Menge<br />

Prozent Menge<br />

64 % 2000 ml 125 % 550 kg<br />

4 % 125 ml<br />

25 % 110 kg<br />

100 % 200 € 100 % 3125 ml 100 % 440 kg<br />

2<br />

Berechne den Grundwert im Rechenschema.<br />

a) Die Partei hat 7 % Mitglieder<br />

verloren. Das sind 630<br />

Personen.<br />

b) Der Preis wurde um 12 %<br />

gesenkt. Das sind 6,60 €.<br />

9000<br />

·____<br />

​ 100<br />

7 ​<br />

630<br />

55 €<br />

·____<br />

​ 100<br />

12 ​ 6,60 €<br />

c) Das Brett wurde um 56 %<br />

gekürzt. Das sind 49 cm.<br />

87,5 cm<br />

·____<br />

​ 100<br />

56 ​ 49 cm<br />

∙100<br />

: 7<br />

90<br />

Die Partei hatte jjj 9000<br />

Mitglieder.<br />

· 100<br />

: 12<br />

0,55 €<br />

Der Preis betrug vorher<br />

55 €.<br />

· 100<br />

: 56<br />

0,875 cm<br />

Das Brett war vorher<br />

87,5 cm lang.<br />

3<br />

Vervollständige das Beispiel. Berechne die Grundwerte wie im Beispiel.<br />

88 % entsprechen 1100 l. a) 19 % entsprechen 627 ml.<br />

·<br />

1250 l ⎯⎯→ 0,88<br />

· 0,19<br />

1100 l<br />

⎯⎯→⎯⎯→jjj<br />

: 0,88<br />

3300 ml<br />

627 ml<br />

: 0,19<br />

⎯⎯→<br />

jjj<br />

b) 101 % entsprechen 324,12 €. c) 63 % entsprechen 35,28 m 2 .<br />

· 1,01<br />

· 0,63<br />

≈ 320,91 € ⎯⎯→⎯⎯→jjj<br />

324,12 € 56 m ⎯⎯→⎯⎯→jjj<br />

2 35,28 m 2<br />

: 1,01<br />

: 0,63<br />

jjj<br />

jjj<br />

4<br />

Bestimme den Grundwert. Wähle selbst einen Rechenweg, den du für sinnvoll hältst.<br />

a) 12 % einer Länge entsprechen<br />

72 cm.<br />

b) Die Partei hat 6 % ihrer<br />

Stimmen verloren. Das sind<br />

3000.<br />

c) Peter hat seine Arbeitszeit um<br />

35 % überschritten. Das sind<br />

14 Stunden.<br />

600 cm 50 000 Stimmen 40 Stunden<br />

19


Prozent- und Zinsrechnung<br />

Prozentsatz – vermehrter und verminderter Grundwert<br />

<br />

2<br />

Bestimme die Prozentsätze wie im Beispiel. Runde wenn nötig wie im Beispiel.<br />

12 € von 33 € a) 18 von 123 Mitgliedern b) 7 von 97 Angestellten<br />

___ 12 j<br />

33 ≈ 0,364 = 36,4 % ______<br />

18<br />

0,146 14,6<br />

______ j 7<br />

0,072 7,2<br />

123<br />

97<br />

j ≈ jjj = jj %<br />

j ≈ jjj = jj %<br />

c) 55 € von 56 € d) 47 von 233 Schülern e) 37 € von 33 €<br />

______ j 55<br />

98,2<br />

______ j 47<br />

0,202 20,2<br />

______ j 37<br />

1,121 112,1<br />

56<br />

233<br />

33<br />

j ≈ jjj = jj %<br />

j ≈ jjj = jj %<br />

j ≈ jjj = jj %<br />

Häufig sind die Prozentwerte verminderte oder vermehrte Grundwerte. Berechne.<br />

a) Eine Hose kostet mit 19 %<br />

Mehrwertsteuer 47,60 €.<br />

Wie viel kostet die Hose<br />

ohne Mehrwertsteuer?<br />

b) Im letzten Jahr stieg die<br />

Mitgliederzahl des Vereins<br />

um 12 %. Er hat nun<br />

560 Mitglieder. Wie viele<br />

Mitglieder hatte er vorher?<br />

c) In der Mediawelt-Meyer gibt<br />

es auf alle Fernseher 12 %<br />

Rabatt. Der Colorstar-760<br />

kostet jetzt 429,76 €. Wie<br />

viel kostet der Fernseher<br />

ohne Rabatt?<br />

Prozentsatz Preis<br />

Prozentsatz Mitgliederzahl Prozentsatz Preis<br />

119 % 47,60 €<br />

1 %<br />

jjj 0,40 €<br />

100 %<br />

jjj 40 €<br />

112 % 560<br />

2 % 10<br />

100 % 500<br />

88 % 429,76 €<br />

1 % ≈ 4,8836 €<br />

100 % ≈ 488,36 €<br />

Die Hose kostet 40 €<br />

ohne Mehrwertsteuer.<br />

d) Beim Roman von Henriette<br />

Minkel sind im Preis 7 %<br />

Mehrwertsteuer enthalten.<br />

Er kostet 14,98 €. Wie viel<br />

kostet der Roman ohne<br />

Mehrwertsteuer?<br />

Prozentsatz Preis<br />

107 % 14,98 €<br />

1 % 0,14 €<br />

Der Verein hatte vorher<br />

500 Mitglieder.<br />

e) Im letzten Jahr ist der Preis<br />

für Milli-Milch um 2,5 %<br />

gestiegen. Der Liter kostet<br />

jetzt 0,82 €. Wie viel kostete<br />

der Liter Milli-Milch vorher?<br />

Prozentsatz Preis<br />

102,5 % 0,82 €<br />

1 % 0,008 €<br />

Der Colorstar­760<br />

kostete 488,36 €<br />

f) In den letzten Tagen ist<br />

der Wasserspiegel um 24 %<br />

gesunken. Er steht jetzt<br />

bei 2,47 m. wo stand der<br />

Wasserspiegel vorher?<br />

Prozentsatz Höhe<br />

76 % 2,47 m<br />

1 % 0,0325 m<br />

<br />

100 % 14 €<br />

100 % 0,80 €<br />

100 % 3,25 m<br />

Der Roman kostet 14 € Der Liter kostete vorher Der Wasserspiegel<br />

ohne Mehrwertsteuer. 0,80 €.<br />

stand bei 3,25 m.<br />

Berechne aus den vermehrten oder verminderten Grundwerten die ursprünglichen Preise. Orientiere<br />

dich an dem Beispiel. Runde auf ganze Cent.<br />

Der Preis ist um 17 %<br />

gestiegen. Er beträgt jetzt<br />

22,50 €.<br />

· 1,17<br />

⎯⎯→<br />

≈ 19,23 € 22,50 €<br />

: 1,17<br />

⎯⎯→<br />

a) Der Preis ist um 25 % gestiegen.<br />

b) Der Preis beinhaltet 19 %<br />

Er beträgt jetzt 120 €. Mehrwertsteuer. Er<br />

beträgt<br />

96 €<br />

· 1,25<br />

⎯⎯→⎯⎯→jjj<br />

jjj : 1,25<br />

3,99 €.<br />

120 € ≈ 3,35 €<br />

· 1,19<br />

⎯⎯→⎯⎯→jjj<br />

jjj : 1,19<br />

3,99 €<br />

20


Prozent- und Zinsrechnung<br />

Geld und Prozente 1<br />

1<br />

Ergänze die Tabelle.<br />

Zinssatz<br />

Kapitel<br />

2 % 4 % 3,5 % 1,5 % 2,7 % 10 % 8 % 2,5 %<br />

45 000 € 900 € 1800 € 1575 € 675 € 1215 € 4500 € 3600 € 1125 €<br />

3000 € 60 € 120 € 105 € 45 € 81 € 300 € 240 € 75 €<br />

≈ ≈ ≈ ≈<br />

170 €<br />

11 333,33 € 226,67 € 453,33 € 396,67 €<br />

306 € 1133,33 € 906,67 € 283,33 €<br />

6600 € 132 € 264 € 231 € 99 € 178,20 € 660 € 528 € 165 €<br />

11 000 € 220 € 440 € 385 € 165 € 297 € 1100 € 880 € 275 €<br />

2<br />

Harald möchte 12 000 € bei einer Bank anlegen.<br />

Angebot 1:<br />

Die Spar-Eifrig-Bank bietet Harald<br />

1 % Jahreszins für die ersten 2000 €<br />

und 4 % für den restlichen Betrag.<br />

Angebot 2:<br />

Die Kundentreu-Bank bietet ihm<br />

3,5 % Zinsen für das ganze Kapital.<br />

a) Welches Angebot sollte Harald wählen, wenn er sein Geld für ein Jahr anlegen möchte?<br />

Angebot 1: 12 420 €; Angebot 2: 12 420 €; Es ist egal.<br />

b) Welches Angebot sollte Harald wählen, wenn er sein Geld für mehr als ein Jahr anlegen möchte?<br />

2 Jahre: Ang. 1: 12 856,80 €; Ang. 2: 12 854,70 €; Harald sollte Angebot 1 wählen.<br />

3<br />

Marianne und Michael Grünberg möchten ein Haus bauen. Für die Zinsen des Baukredits können<br />

sie pro Jahr maximal 7000 € aufbringen. Die Vertragsbedingungen weisen einen Zinssatz von 5,6 %<br />

Zinsen aus.<br />

a) Wie hoch kann die Kreditsumme höchstens sein?<br />

Die Kreditsumme kann höchstens 125 000 € betragen.<br />

b) Von den 7000 € sollen bereits 2100 € für die Abzahlung des Kredits in ersten Jahr zur Verfügung<br />

stehen. Wie hoch darf die Kreditsumme bei obigem Zinssatz jetzt sein?<br />

Die Kreditsumme kann höchstens 87 500 € betragen.<br />

4<br />

Familie Dieterle hat 35 000 € geerbt. Sie legen das Geld bei einer Bank zu einem Zinssatz von 4,5 %<br />

an.<br />

a) Untersuche, wie sich das Kapital in den nächsten 10 Jahren entwickelt. Runde auf ganze Cent.<br />

Jahre 1 2 3 4 5 6 7 8 9 10<br />

Kapital 36 575 € 39 940,81 € 43 616,37 € 47 630,16 € 52 013,33 €<br />

38 220,88 € 41 738,15 € 45 579,10 € 49 773,52 € 54 353,93 €<br />

b) Wie entwickelt sich das Kapital, wenn die Familie am Jahresende jeweils 500 € abhebt?<br />

Jahre 1 2 3 4 5 6 7 8 9 10<br />

Kapital<br />

36 075 € 38 372,30 €<br />

37 198,38 €<br />

40881,01 € 43 620,59 € 46612,27 €<br />

39 599,06 € 42 220,66 € 45 083,51 € 48 209,83 €<br />

21


Prozent- und Zinsrechnung<br />

Geld und Prozente 2<br />

<br />

Verzinse jährlich mit dem angegebenen Prozentsatz. Brich die Rechnung ab, wenn sich das Kapital<br />

ungefähr verdoppelt hat. Trage die Verdopplungszeit in die Tabelle ein.<br />

a) Kapital 1000 € Zinssatz 8 % b) Kapital 2000 € Zinssatz 8 % c) Kapital 500 € Zinssatz 4 %<br />

d) Kapital 700 € Zinssatz 7 % e) Kapital 1000 € Zinssatz 9 % f) Kapital 500 € Zinnssatz 3 %<br />

a) b) c) d) e) f)<br />

Prozentsatz 8 % 8 % 4 % 7 % 9 % 3 %<br />

Verdopplungszeit ca. 9 Jahre ca. 9 Jahre ca. 18 Jahre ca. 10 Jahre ca. 8 Jahre 23 – 24 Jahre<br />

2<br />

Berechne das Kapital nach mehreren Jahren. Runde auf volle Cent-Beträge.<br />

Ein Kapital von 4000 € wird<br />

3 Jahre mit einem Prozentsatz<br />

von 5 % verzinst.<br />

4000 €<br />

· 1,05 3<br />

⎯⎯→<br />

4630,50 €<br />

a) Ein Kapital von 3600 € wird b) Ein Kapital von 1000 € wird<br />

5 Jahre mit einem Zisnsatz 7 Jahre mit einem Zinssatz<br />

von 3 % verzinst.<br />

von 4,5 % verzinst.<br />

jjj · 1,03 5<br />

jjj<br />

3600 € ⎯⎯→ ≈ 4173,39 € 1000€ · 1,0457 ⎯⎯→ ≈ 1360,86 €<br />

<br />

4<br />

Ein Kapital wurde 6 Jahre mit einem Zinssatz von 4 % verzinst. Es sind nun 11 111 € vorhanden.<br />

Welches Kapital wurde angelegt?<br />

Ergänze die Lücken in den Tabellen. Der Jahreszinssatz gilt jeweils für die ganze Tabelle.<br />

a) Zinssatz 4 % b) Zinssatz 3 %<br />

Kapital<br />

Zeit<br />

jjj<br />

≈ 8781,18 € · 1,046<br />

11 111 €<br />

jjj⎯⎯→<br />

: 1,04 6<br />

⎯⎯→<br />

Es wurde ein Kapitel von 8781,18 € angelegt.<br />

1__<br />

4 Jahr 2 Monate 7 Monate Zeit<br />

Kapital<br />

1__ Jahr 10 Monate 1 Monat<br />

3<br />

15 000 € 150 € 100 € 350 € 4800 € 48 € 120 € 12 €<br />

7500 € 75 € 50 € 175 € 2 000 € 20 € 50 € 5 €<br />

1200 € 12 € 8 € 28 € 1750 € 17,50 € 43,75 € ≈ 4,38 €<br />

c) Zinssatz 1,8 % d) Zinssatz jj 2 %<br />

Zeit<br />

Kapital<br />

10 Tage 15 Tage 60 Tage<br />

Zeit<br />

Kapital<br />

1 Tag 9 Tage 2 Monate<br />

10 000 € 5 € 7,50 € 30 € 7500 € ≈ 0,42 € 3,75 € 25 €<br />

2000 € 10 € 15 € 60 € 9000 € 0,50 € 4,50 € 30 €<br />

4400 € 2,20 € 3,30 € 13,20 € 1500 € ≈ 0,08 € 0,75 € 5 €<br />

22


Prozent- und Zinsrechnung<br />

Vermischte Übungen 1<br />

1<br />

Für zwei Werbewochen senkt ein Geschäft die Preise zunächst um 25 %. Danach setzt es die Preise<br />

für die Aktion um 25 % wieder herauf.<br />

a) Ein Artikel kostete ursprünglich 164,80 €. Wie viel kostete er in den Werbewochen, wie viel nach<br />

den Werbewochen?<br />

In den Werbewochen: Der Artikel kostet 123,60 €.<br />

Nach den Werbewochen: Der Artikel kostet 154,50 €.<br />

b) Um wie viel Prozent hat sich der Preis insgesamt geändert?<br />

Der Preis ist um 6,25 % gestiegen.<br />

2<br />

Nuss-Nougat-Creme besteht zu ca. 33 % aus Fett, zu ca. 55 % aus Kohlenhydraten und zu ca. 8 %<br />

aus Eiweiß. 4 % sind sonstige Bestandteile.<br />

a) Berechne die ungefähren Mengen in einem 800 g Glas.<br />

264 g Fett 440 g Kohlenhydrate 64 g Eiweiß 32 g sonstige Bestandteile<br />

b) Leopold liebt Nuss-Nougat-Creme. Er weiß aber auch, dass man täglich nur etwa 60 g Fett<br />

zu sich nehmen soll. Wie viel Creme darf Leopold pro Tag essen, wenn er höchstens ​ 1__<br />

4 ​seines<br />

Fettbedarfs mit Nuss-Nougat-Creme decken möchte? Wie lange reicht dann sein 800-g-Glas?<br />

Er darf höchstens 45,45 g Nuss-Nougat-Creme pro Tag essen.<br />

Ein 800 g-Glas reicht für fast 18 Tage.<br />

3<br />

Alexander kauft im April beim Händler Herrmann Tech einen Computer für 420 €. Dieser hätte im<br />

Februar noch 525 € gekostet. Herrmann Tech hat im Großhandel 315 € bezahlt.<br />

Welche Aussagen sind richtig? Finde das Lösungswort. richtig falsch<br />

„Der Preis des Computers wurde zum April um 20 % gesenkt.“ R U<br />

„Erhöht der Händler den neuen Preis um 20 %, so erhält man wieder 525 €.“ T O<br />

„Von den 420 € kann Herrmann Tech 33,3 % als Gewinn verbuchen.“ I T<br />

„Der neue Preis beträgt 80 % des alten Preises.“ E A<br />

„Herrmann Tech hat den Computer 33,3 % über seinem Einkaufspreis verkauft.“ S M<br />

„Der Preis im Februar beträgt 125 % des neuen Preises.“ O N<br />

„Hätte Herrmann Tech den Computer für 525 € verkauft, so hätte sein Gewinn<br />

über 50 % des Verkaufspreises gelegen.“<br />

Lösungswort: ROSE <br />

A<br />

M<br />

4<br />

In einer Klasse haben 12 Schüler blaue Augen, 20 % aller Schüler haben braune Augen, 40 % aller<br />

Schüler haben weder blaue noch braune Augen. Die Hälfte aller Mädchen trägt eine Brille, 6 Jungen<br />

sind Einzelkinder, 60 % aller Schüler sind Mädchen, in der Klasse sind 12 Brillenträger, ​ 1__ ​aller Schüler<br />

3<br />

sind Einzelkinder.<br />

a) Wie viele Schüler sind in der Klasse? Es sind 30 Schüler in der Klasse.<br />

b) Wie viele Mädchen bzw. Jungen sind in der Klasse? Es sind 18 Mädchen und 12 Jungen in der Klasse.<br />

c) Wie viel Prozent der Jungen tragen eine Bille? 25 % der Jungen trägt eine Brille.<br />

__<br />

d) Wie viel Prozent der Mädchen sind Einzelkinder? Es sind 22,​2​ % der Mädchen Einzelkinder.<br />

23


Prozent- und Zinsrechnung<br />

Vermischte Übungen 2<br />

<br />

Frau Ruppig und der Ladenbesitzer Herr Friedbert haben Streit. Frau Ruppig möchte eine Bohrmaschine<br />

kaufen. Diese ist mit 180 € ausgezeichnet. Hinzu kommen noch 19 % Mehrwertsteuer. Da<br />

die Maschine am Gehäuse eine kleine Beschädigung hat, will Herr Friedbert 25 % Rabatt geben.<br />

Herr Friedbert zieht zuerst 25 % ab und berechnet dann den Preis mit Mehrwertsteuer. Frau Ruppig<br />

verlangt, dass er umgekehrt vorgeht. Was meinst du dazu?<br />

Herr Friedberts Rechenweg<br />

Frau Ruppigs Rechenweg<br />

Preis nach Rabatt Preis mit Mehrwertsteuer Preis mit Mehrwertsteuer Preis nach Rabatt<br />

135 € 160,65 € 214,20 € 160,65 €<br />

Der Streit ist nicht nötig. Es ist egal, wie man vorgeht.<br />

2<br />

Familie Petersen besitzt ein Grundstück. Es<br />

hat eine Länge von 30 m und eine Breite von<br />

40 m. Durch den Ausbau zweier Straßen wird<br />

das Grundstück um 10 % kürzer und um 15 %<br />

schmaler.<br />

a) Berechne die neuen Seitenlängen des<br />

Grundstücks. 27 m, 34 m<br />

b) Zeichne das ursprüngliche Grundstück<br />

auf der rechten Seite. Zeichne die neuen<br />

Grundstücksgrenzen ein.<br />

c) Um wie viel Prozent ist die Fläche des<br />

Grundstücks kleiner geworden? 23,5 %<br />

d) Markiere anhand der Zeichnung,<br />

warum das Grundstück um weniger als<br />

15 % + 10 % = 25 % kleiner geworden ist.<br />

30 m<br />

40 m<br />

Dieses Stück kann nur einmal<br />

weggenommen werden<br />

(1,5 % der Gesamtfläche).<br />

10 m<br />

<br />

Frau Kleingeld verdient im Moment 2500 € im Monat. Ihr Chef ist mit ihrer Arbeit mehr als zufrieden,<br />

deshalb bietet er ihr als Gehaltserhöhung zwei Angebote zur Auswahl.<br />

Angebot 1: Das Monatsgehalt steigt in den nächsten 3 Jahren zu Jahresbeginn um jeweils 270 €.<br />

Angebot 2: Das Monatsgehalt steigt in den nächsten 3 Jahren zu Jahresbeginn um jeweils 10 %.<br />

Welches Angebot ist langfristig für Frau Kleingeld lohnender?<br />

Angebot 1: 3310 € Angebot 2: 3327,50 €<br />

Angebot 2 ist langfristig lohnender.<br />

4<br />

Nach einer Preissenkung um 8 % verkauft ein Warenhaus seine Artikel für unten stehende Preise.<br />

Berechne jeweils den ursprünglichen Preis. Runde auf ganze Cent.<br />

neuer Preis 45,90 € 137,50 € 257 € 17,80 € 89,90 € 2,38 € 78,30 €<br />

alter Preis<br />

≈<br />

49,89 €<br />

≈<br />

149,46 €<br />

≈<br />

279,35 €<br />

≈<br />

19,35 €<br />

≈<br />

97,72 €<br />

≈<br />

2,59 €<br />

≈<br />

85,11 €<br />

24


Prozent- und Zinsrechnung<br />

Vermischte Übungen 3<br />

1<br />

Hannes kauft im Computerfachgeschäft von Herrn Bit einen Computer, der bereits ein halbes Jahr<br />

alt ist. Herr Bit gewährt Hannes deshalb 20 % Rabatt. Da Hannes aber nicht genug Geld hat, lässt<br />

sich Herr Bit auf eine Ratenzahlung ein, für die er aber einen Aufschlag von 5 % verlangt. Hannes<br />

bezahlt nun insgesamt 840 €.<br />

a) Wie viel hätte Hannes bezahlen müssen,<br />

wenn er nicht auf die Ratenzahlung angewiesen<br />

wäre?<br />

b) Wie viel hat der Computer vor der Preissenkung<br />

gekostet?<br />

840 € : 1,05 = 800 €<br />

800 € : 0,8 = 1000 €<br />

Hannes hätte 800 € bezahlen müssen. Der Computer hat 1000 € vor der<br />

<br />

Preissenkung <br />

gekostet.<br />

c) Wie viel Prozent hat Hannes trotz seiner Ratenzahlung gespart?<br />

_____ ​<br />

<br />

160 ​ = 16 % Hannes hat 16 % gespart.<br />

1000<br />

2<br />

3<br />

Reiner Apfelsaft soll im Verhältnis 2 zu 3 zu Apfelschorle verdünnt werden. Die Firma Bertram<br />

möchte 10 000 l Apfelschorle herstellen. Wie viel Apfelsaft muss sie im Großhandel einkaufen?<br />

​__<br />

<br />

2 ​· 10 000 l = 4000 l Die Firma Bertram muss 4000 l Apfelsaft einkaufen.<br />

5<br />

Henriette ist gerade 13 Jahre alt und hat 2000 € von ihrem Lieblingsonkel geschenkt bekommen.<br />

Die Bank bietet 4 % Zinsen. Zu ihrem 18. Geburtstag möchte sie 2200 € für ihren Führerschein zur<br />

Verfügung haben.<br />

a) Wie viel Geld muss Henriette jetzt mindestens anlegen, um ihr Sparziel zu erreichen?<br />

2200 € : 1,04 5 ≈ 1808,24 €<br />

Antwort: Henriette muss mindestens 1808,24 € anlegen.<br />

b) Welchen Anteil ihres Geschenks kann Henriette dann trotzdem noch im Sommerurlaub verjubeln?<br />

______ ​ 191,76 ​≈ 9,59 % Henriette kann ungefähr 9,59 % des Geschenks verjubeln.<br />

2200<br />

4<br />

5<br />

Siegfried hat 16 000 € bei einer Bank angelegt. Bereits nach 4 Monaten hat er es sich anders überlegt<br />

und lässt sich sein Geld mit Zinsen wieder auszahlen. Er erhält 16 160 €. Welchen Jahreszinssatz<br />

hat die Bank bezahlt? _______ ​ 3 · 160 € ​ Die Bank hat einen Zinsatz von 3 % bezahlt.<br />

<br />

16 000 €<br />

Petra Pasulke bezahlte ursprünglich 250 € Miete für ihr Studentenzimmer, nun sie ist sauer. In ihrem<br />

Mietvertrag steht, dass ihr Vermieter Karl Reibach die Miete in drei Jahren um maximal 15 % erhöhen<br />

darf. Herr Reibach hat die Miete im ersten, zweiten und dritten Jahr um jeweils 5 % erhöht.<br />

a) Wie viel Miete muss sie jetzt zahlen? Sie muss jetzt etwa 289,41 € Miete zahlen.<br />

b) Hat sich der Vermieter an den Mietvertrag gehalten? Nein, die Obergrenze liegt bei 287,50 €.<br />

25


Winkel und besondere Linien<br />

Winkelsätze an Geradenkreuzungen 1<br />

<br />

Gib in den Skizzen an, wie groß die gesuchten Winkel sein müssten. Messen funktioniert nicht. Die<br />

grünen Geraden sind jeweils parallel zueinander.<br />

a)<br />

Begründung<br />

γ<br />

α = 80° , α ist Nebenwinkel zu 100°.<br />

100°<br />

<br />

α<br />

= 100° , ist Scheitelwinkel zu 100°.<br />

γ = 100° , γ ist Stufenwinkel<br />

zu 100°.<br />

b)<br />

c)<br />

<br />

γ<br />

95°<br />

α<br />

40°<br />

α<br />

γ<br />

<br />

α = 40° = 140° γ = 40° α = 95° = 95° γ = 85°<br />

d)<br />

70°<br />

<br />

γ<br />

α<br />

e)<br />

α<br />

130°<br />

γ<br />

65°<br />

<br />

α = 110° = 70° γ = 70° α = 50° = 50° γ = 115°<br />

f)<br />

g)<br />

γ<br />

α<br />

135°<br />

50°<br />

α<br />

<br />

105°<br />

<br />

60°<br />

γ<br />

α = 45° = 75° γ = 105° α = 70° = 60° γ = 110°<br />

26


Winkel und besondere Linien<br />

Winkelsätze an Geradenkreuzungen 2<br />

1<br />

Gib in den Skizzen an, wie groß die gesuchten Winkel sein müssten. Die grünen Geraden sind<br />

jeweils parallel zueinander.<br />

a)<br />

b)<br />

α<br />

α<br />

<br />

γ<br />

40°<br />

<br />

γ<br />

38°<br />

33°<br />

α = 140° = 40° γ = 140° α = 33° = 147° γ = 142°<br />

2<br />

In einem Parallelogramm werden durch die<br />

Diagonale die Winkel α und γ zerlegt. Welche<br />

Winkel sind gleich groß? Begründe.<br />

α<br />

1<br />

= γ 1<br />

und α 2<br />

= γ 2<br />

;<br />

da jeweils Wechselwinkel zueinander.<br />

γ 2<br />

γ 1<br />

α 1<br />

α 2<br />

3<br />

In einem Trapez gilt α = 80° und γ = 135°.<br />

Berechne die anderen Winkelgrößen.<br />

δ<br />

γ<br />

= 45° δ = 100°<br />

α<br />

<br />

4<br />

Welches der Vierecke ist ein Trapez? Begründe.<br />

a)<br />

70°<br />

b)<br />

135°<br />

72°<br />

43°<br />

Kein Trapez, da keine parallelen Seiten.<br />

Kein Trapez, da die Winkel sich nicht<br />

<br />

zu 180° ergänzen.<br />

c)<br />

d)<br />

40°<br />

115° 125°<br />

115°<br />

110° 70°<br />

65°<br />

Ist ein Trapez, da die entsprechenden<br />

Kein Trapez, da sich der 70° Winkel und<br />

Winkel sich zu 180° ergänzen.<br />

der 115° Winkel nicht zu 180° ergänzen.<br />

27


Winkel und besondere Linien<br />

Winkelsätze an Vielecken 1<br />

<br />

Gib in den Skizzen an, wie groß die gesuchten Winkel sein müssten.<br />

a)<br />

b)<br />

α<br />

113°<br />

36°<br />

<br />

36°<br />

γ<br />

25°<br />

42°<br />

α<br />

110°<br />

α = 67°<br />

α = 110° = 144° γ = 34°<br />

c)<br />

100°<br />

d)<br />

α<br />

80°<br />

20°<br />

<br />

α<br />

65°<br />

35°<br />

35°<br />

55°<br />

e)<br />

α = 115°<br />

α = 70° = 35°<br />

g<br />

h<br />

34°<br />

129°<br />

α<br />

17° f)<br />

g i h<br />

γ<br />

34°<br />

30°<br />

<br />

α<br />

45°<br />

50°<br />

α = 51°<br />

α = 85° = 85° γ = 65°<br />

g)<br />

h) Regelmäßiger Stern<br />

59°<br />

α<br />

31°<br />

α<br />

<br />

<br />

α = 31° = 59°<br />

α = 60° = 120°<br />

28


Winkel und besondere Linien<br />

Winkelsätze an Vielecken 2<br />

1<br />

Gib in den Skizzen an, wie groß die gesuchten Winkel sein müssten.<br />

a)<br />

b)<br />

105°<br />

γ<br />

110°<br />

α<br />

<br />

α<br />

c)<br />

α = 70°<br />

α = 75° = 75° γ = 105°<br />

d)<br />

g<br />

56°<br />

35°<br />

α α<br />

g i h<br />

30°<br />

α<br />

40°<br />

56°<br />

<br />

<br />

α<br />

h<br />

α = 255°<br />

α = 62° = 59°<br />

2<br />

b i d<br />

c i e<br />

87°<br />

e<br />

ε<br />

α<br />

a<br />

155°<br />

d<br />

b<br />

87°<br />

c<br />

γ<br />

87°<br />

α = 112°<br />

γ = 93°<br />

ε = 93°<br />

3<br />

4<br />

In einem rechtwinkligen Dreieck ist der größte Winkel zehnmal so groß wie der kleinste. Gib alle<br />

Winkelgrößen an.<br />

α = 9° = 81° γ = 90°<br />

Ein Giebeldach hat die Dachneigung α und die<br />

γ<br />

Dachkanten stehen in einem Winkel γ zueinander.<br />

α α<br />

a) Welchen Winkel schließen die Dachkanten ein, wenn die Neigungswinkel 22° betragen?<br />

γ = 180° – 2 · α = 136°<br />

b) Bestimme die Dachneigung, wenn die Dachkanten einen Winkel von 110° einschließen.<br />

2 α = 180° – 110° = 70°. Somit α = 35°.<br />

c) Welche Winkel der Dachkanten können entstehen, wenn nur Dachneigungen zwischen 20° und<br />

60° zugelassen sind?<br />

Die Dachkanten schließen dann einen Winkel zwischen 60° und 140° ein.<br />

29


Winkel und besondere Linien<br />

Mittelsenkrechte – Lot – Winkelhalbierende – Mittelparallele 1<br />

<br />

Konsturiere mit Zirkel und Lineal die Mittelsenkrechte zu ___<br />

AB . Bringe dazu zunächst die<br />

Konstruktionsschritte in die richtige Reihenfolge.<br />

3 1 4 2<br />

Schritt j Schritt j Schritt j Schritt j<br />

Kennzeichne die<br />

Schnittpunkte S 1<br />

und<br />

S 2<br />

der beiden Kreise.<br />

Zeichne um A einen<br />

Kreis K 1<br />

mit Radius<br />

r > 1__ ___<br />

AB .<br />

2<br />

A<br />

Zeichne die Gerade<br />

durch S 1<br />

und S 2<br />

.<br />

Zeichne um B einen<br />

Kreis K 2<br />

mit dem gleichen<br />

Radius r.<br />

S 1<br />

S 2<br />

B<br />

2<br />

Bringe die Bilder in die richtige Reihenfolge. Was wurde konstruiert?<br />

a) Schritt j2 Schritt j4 Schritt j3 Schritt j1<br />

P<br />

P<br />

P<br />

P<br />

g<br />

g<br />

g<br />

B<br />

B<br />

B<br />

A<br />

A<br />

A<br />

Q<br />

Q<br />

Was wurde konstruiert? Lot von P auf g<br />

b) Schritt j4 Schritt j1 Schritt j3 Schritt j2<br />

g<br />

g<br />

g i h<br />

g<br />

g<br />

g<br />

h<br />

h<br />

h<br />

h<br />

Was wurde konstruiert? Mittelparallele<br />

<br />

Zeichne einen 80° großen Winkel und halbiere ihn mit Zirkel und Lineal.<br />

0


Winkel und besondere Linien<br />

Mittelsenkrechte – Lot – Winkelhalbierende – Mittelparallele 2<br />

1<br />

2<br />

Wo liegen alle Punkte, die<br />

a) von zwei Punkten A und B gleich weit entfernt sind? Mittelsenkrechten<br />

b) von den Schenkeln a, b eines Winkels gleichen Abstand haben? Winkelhalbierenden<br />

<br />

c) zu zwei Parallelen g und h gleichen Abstand haben? Mittelparallele<br />

In welchem Punkt S schneiden sich die Mittelsenkrechten der Strecken ​ ___<br />

AB​und ​ ___<br />

CD​?<br />

6<br />

5<br />

y<br />

S<br />

A (1 | 1)<br />

B (4 | 0)<br />

C (5,5 | 0,5)<br />

D (8,5 | 3,5)<br />

4<br />

3<br />

D<br />

4 5<br />

S (jj | jj )<br />

2<br />

1<br />

A<br />

B<br />

1 2 3 4 5 6 7 8 9<br />

C<br />

x<br />

3<br />

Konstruiere einen Punkt P, der von den Geraden g und h jeweils 2 cm entfernt ist. Beschreibe dein<br />

Vorgehen.<br />

g<br />

Beschreibung:Konstruiere jeweils mithilfe zweier<br />

P<br />

Senkrechten eine Parallele zu den Schenkeln<br />

im Abstand von 2 cm; der Schnittpunkt der<br />

beiden Parallelen ist P. (Oder: eine Parallele<br />

h<br />

und die Winkelhalbierende)<br />

4<br />

In einen Kreis sind zwei Sehnen eingezeichnet. Zeichne zu den beiden Sehnen jeweils die<br />

Mittelsenkrechte.<br />

M<br />

Was wird konstruiert? Mittelpunkt <br />

des Kreises.<br />

31


Winkel und besondere Linien<br />

Besondere Linien und Punkte im Dreieck 1<br />

<br />

Konstruiere den Inkreis des Dreiecks. Gib den Mittelpunkt M I und den Radius r I des Inkreises an.<br />

Runde auf eine Stelle nach dem Komma.<br />

y<br />

6<br />

C<br />

5<br />

D<br />

F<br />

a) A (3 | 0); B (5 | 2); C (0 | 5)<br />

M I (jj 3,2 | jj) 1,8<br />

r I = 1,1 cm<br />

4<br />

3<br />

2<br />

1<br />

A<br />

M I<br />

B<br />

E<br />

1 2 3 4 5 6 7 8 9 10<br />

M I<br />

x<br />

b) D (2 | 6); E (7 | 1); F (10 | 5)<br />

M I (jj 6,8 | jj) 3,7<br />

r I = 1,7 cm<br />

2<br />

Konstruiere den Umkreis und den Inkreis des Dreiecks ABC mit A (1 | 3); B (6 | 1); C (2 | 5). Gib die<br />

Mittelpunkte M U und M I sowie die Radien r U und r I an. Runde auf eine Stelle nach dem Komma.<br />

Umkreis<br />

y<br />

5<br />

4<br />

3<br />

2<br />

C<br />

A<br />

M U (jj 3,8 | jj) 2,8<br />

r U = 2,8 cm<br />

M<br />

Inkreis<br />

I<br />

M<br />

M I<br />

U<br />

(jj 2,3 | jj) 3,5<br />

r I = 0,9 cm<br />

1<br />

1 2 3 4 5 6 7 8 9 10<br />

B<br />

x<br />

<br />

a) Welcher Punkt P ist von den Eckpunkten des Dreiecks ABC gleich weit entfernt?<br />

b) Welcher Punkt Q ist von den Seiten des Dreiecks ABC gleich weit entfernt?<br />

y<br />

4<br />

3<br />

2<br />

1<br />

C<br />

Q<br />

A<br />

P<br />

B<br />

x<br />

1 2 3 4 5 6 7 8 9 10<br />

A (1,5 | 2); B (7,5 | 1);<br />

C (6,5 | 4,5)<br />

Gib auch an, welche<br />

Linien du für die<br />

Konstruktion der Punkte P<br />

und Q benutzt.<br />

a) P (jj 4,6 | jj) 2,1<br />

Mittelsenkrechten<br />

b) Q (jj 5,7 | jj) 2,6<br />

Winkelhalbierenden<br />

2


Winkel und besondere Linien<br />

Besondere Linien und Punkte im Dreieck 2<br />

1<br />

Bestimme den Schwerpunkt S des Dreiecks. Runde auf eine Stelle nach dem Komma.<br />

y<br />

5<br />

C<br />

4<br />

3<br />

2<br />

1<br />

A<br />

F<br />

E<br />

S<br />

S<br />

B<br />

D<br />

x<br />

a) A (0 | 1); B (6 | 1); C (0 | 4)<br />

2 2<br />

S (jj | jj)<br />

b) D (8 | 1); E (9,5 | 4,5);<br />

F (2 | 5)<br />

S (jj 6,5 | jj) 3,5<br />

1 2 3 4 5 6 7 8 9 10<br />

2<br />

Konstruiere den Höhenschnittpunkt H des Dreiecks.<br />

y<br />

C F<br />

5<br />

a) A (0 | 2); B (3 | 0); C (2 | 5)<br />

H (jj 0 | jj) 2<br />

4<br />

b) D (5 | 2); E (8 | 2); F (4 | 5)<br />

3<br />

A<br />

2<br />

H<br />

D<br />

E<br />

H (jj 4 | jj) 0,7<br />

1<br />

B<br />

H<br />

1 2 3 4 5 6 7 8 9 10<br />

x<br />

3<br />

Konstruiere im Dreieck ABC mit A (2 | 2); B (9 | 4); C (6,5 | 7) den Schnittpunkt M der Mittelsenkrechten,<br />

W der Winkelhalbierenden, S der Seitenhalbierenden, H der Höhengeraden.<br />

y<br />

C<br />

Welcher der Punkte ist<br />

7<br />

welcher Schnittpunkt?<br />

H<br />

6<br />

jj W (6,4 | 4,7)<br />

jj H (6,8 | 6,0)<br />

5<br />

W<br />

S<br />

jj S (5,9 | 4,4)<br />

4<br />

B<br />

M<br />

jj M (5,3 | 3,6)<br />

3<br />

2<br />

A<br />

1<br />

x<br />

1 2 3 4 5 6 7 8 9 10<br />

33


Winkel und besondere Linien<br />

Besondere Linien und Punkte im Dreieck 3<br />

<br />

Konstruiere die gesuchten Punkte.<br />

a) Drei Polarstationen P 1 ,<br />

P 3<br />

P 2 und P 3 benötigen ein<br />

P 2<br />

gemeinsames Depot.<br />

P 1<br />

Schnittpunkt der Mittelsenkrechten<br />

b) Auf einer Grünfläche<br />

zwischen den Straßen<br />

soll ein möglichst großes,<br />

kreisförmiges Blumenbeet<br />

angelegt werden.<br />

Winkelhalbierenden<br />

Inkreis<br />

D<br />

c) In einem Wettbewerb<br />

wollen drei Schüler S 1 , S 2<br />

und S 3 einen Ball von verschiedenen<br />

Startpunkten<br />

erreichen. Wohin muss der<br />

Ball gelegt werden, damit<br />

der Wettbewerb fair ist?<br />

Mittelsenkrechten<br />

Mittelpunkt des Umkreises<br />

b)<br />

S 3 S 2<br />

S 1 B<br />

d) Ein Pappdreieck soll auf<br />

einer Nadel balanciert<br />

werden.<br />

Seitenhalbierenden<br />

Schwerpunkt<br />

S<br />

e) Beim Boccia sind die<br />

K<br />

K 3<br />

Kugeln K 1 , K 2 und K 3<br />

gleich weit von der kleinen<br />

K 2<br />

Zielkugel liegen geblieben.<br />

Die kleine Kugel wurde<br />

bereits entfernt.<br />

K 1<br />

Wo hat sie gelegen?<br />

Schnittpunkt der Mittelsenkrechten<br />

f) Aus einer dreieckigen<br />

Metallplatte soll eine<br />

möglichst große<br />

Kreisscheibe aus geschnitten<br />

werden.<br />

Winkelhalbierenden<br />

Inkreis<br />

4


Rationale Zahlen<br />

Anordnung und Betrag an der Zahlengeraden<br />

1<br />

Trage die Zahlen auf der Zahlengeraden ein: – ​ 1__ ​; 2,1; – 0,75 ; – ​3__ ​; 1​3__ ​; – 3 ; – ​33<br />

8 2 4 ___<br />

10 ​<br />

0 1<br />

–___<br />

​ 33<br />

10 ​– 3 – ​3 __<br />

2 ​ – 0,75 – __ ​ 1 1 ​__<br />

3<br />

4 ​ 2,1<br />

8 ​<br />

2<br />

3<br />

Ergänze.<br />

1,8 oder<br />

Zahl 8,5 2,6 – 1,8 – ​ 3__<br />

5 ​ 0 – 0,04 ​__<br />

3<br />

8 ​ __ ​1 3 ​ oder – __ ​ 1 3 ​<br />

Betrag 8,5 2,6 1,8 ​ 3 __<br />

5 ​ 0 0,04 ​ 3 __<br />

8 ​ ​ 1__<br />

3 ​<br />

Spiegelzahl – 8,5 – 2,6 – 1,8 __ ​ 3 5 ​ 0 0,04 – ​ 3__<br />

8 ​ – ​__<br />

1<br />

3 ​ oder __ ​ 1 3 ​<br />

oder 1,8<br />

Wie weit sind die beiden Zahlen auf der Zahlengeraden voneinander entfernt und welche Zahl liegt<br />

in ihrer Mitte?<br />

Zahl 1 Zahl 2 Entfernung Mitte Zahl 1 Zahl 2 Entfernung Mitte<br />

a) + 4 – 3 7 0,5 b) – 2 – 11 9 – 6,5<br />

c) – 2,5 + 1,5 4 – 0,5 d) + 1,4 – 3,8 5,2 – 1,2<br />

e) – ​ 1__<br />

2 ​ – ​3__ 4 ​ ​__<br />

1 4 ​ – ​5 __ ​ f) – ​2__<br />

8 5 ​ – 7,6 7,2 – 4<br />

g) – 10,2 0,6 10,8 – 4,8 h) + 0,7 – 7,9 8,6 – 3,6<br />

4<br />

a) Zeichne den Streckenzug ABCDEFGA:<br />

A (4 | – 4), B (2 | 1,5), C (1 | 0), D (1 | – 3),<br />

E (– 3 | – 3), F (– 2 | – 5), G ​( 2​ 1__<br />

2 ​| – 6 )​.<br />

b) Vertausche nun bei jedem Punkt die<br />

x- und die y-Koordinate und verbinde<br />

die neuen Punkte in der gleichen<br />

Reihenfolge. Was fällt dir auf?<br />

Figur ist gespiegelt.<br />

G 9<br />

A9<br />

3<br />

2<br />

1<br />

− 7 − 6 − 5 − 4 − 3 − 2 − 1 1 2 3 4 5 6 7<br />

− 1<br />

F 9<br />

D 9 C 9<br />

E<br />

F<br />

− 2<br />

− 3<br />

− 4<br />

− 5<br />

− 6<br />

y<br />

C<br />

B 9<br />

B<br />

D<br />

G<br />

A<br />

x<br />

5<br />

Die Frösche Quicks und Quacks sitzen auf der Zahlengeraden:<br />

Quicks bei – 87, Quacks bei + 15.<br />

a) Quacks springt in „3-er“-Sprüngen auf Quicks zu.<br />

Nach wie vielen Sprüngen hat er Quicks erreicht?<br />

34<br />

b) Quicks springt im „2-er“-Takt, Quacks im „3-er“. Wie viele Sprünge wären bis zu einem Treffen<br />

notwendig, wenn beide gleichzeitig starten und aufeinander zuspringen würden?<br />

Quicks 21, Quacks 20<br />

c) Quicks versucht, Quacks auf der Zahlengeraden zu entkommen. An welcher Stelle erreicht ihn<br />

Quacks dennoch?<br />

bei – 291, nach 102 Sprüngen<br />

35


Rationale Zahlen<br />

Addition und Subtraktion<br />

<br />

2<br />

a) Additionstabelle b) Subtraktionstabelle<br />

+ – 4 2,5 – 8,2 3__<br />

5<br />

– 9 – 13 – 6,5 – 17,2 – 8 __ 2<br />

5<br />

1,5 – 2,5 4 – 6,7 2,1<br />

– 7__<br />

2<br />

– 7 1 __<br />

2<br />

– 1 – 11,7 – 2 9 ___<br />

10<br />

– 1 – 5 1,5 – 9,2 – 2 __<br />

5<br />

– + 4 – 2,5 – 8,2 0,8<br />

– 9 – 13 – 6,5 – 0,8 – 9,8<br />

1__<br />

4<br />

– 3 __ 3<br />

4<br />

Berechne im Kopf und addiere dann deine Ergebnisse pro Teilaufgabe.<br />

2 3 __<br />

4<br />

8,45 – 0,55<br />

– 3,5 – 7,5 – 1 4,7 – 4,3<br />

0 – 4 2,5 8,2 – 0,8<br />

a) – 8 – 12 = – 20 b) 2 – 17 = – 15 c) – 8 + 19 = 11 d) 12 – (– 13) = 25<br />

– 1,2 + 3 = 1,8<br />

1__ – 4 __ 3<br />

– 5 =<br />

4<br />

4 – 7 – 8,6 = – 15,6 – 6,3 + (– 9) = – 15,3<br />

2,5 – 1,8 = 0,7<br />

5 – 6,25 = – 1,25 – 1,6 – 5,9 = – 7,5 – 2,2 – 8,8 = – 11<br />

– 3,7 + 6,2 = 2,5 – 5,8 + 8 1__ = 2,7<br />

2<br />

4,1 – (– 1,4) = 5,5<br />

0 –<br />

4__<br />

5 = – 0,8<br />

– 15<br />

– 18,3<br />

– 6,6<br />

– 2,1<br />

Lösungssummen: – 18,3; – 15; – 6,6; – 2,1<br />

jjj jjj jjj jjj<br />

<br />

a) Addiere zu 1 alle Zahlen der Schlange.<br />

(– 3) + 4,5 + (– 8) + (– 13) + __ 3<br />

1 + 2 + (– __ 1<br />

2 ) + 9,5<br />

=<br />

b) Subtrahiere von 1 alle Zahlen der Schlange.<br />

(– 3) – 4,5 – (– 8) – (– 13) – __ 3<br />

1 – 2 – (– __ 1<br />

2 ) – 9,5<br />

=<br />

– 8<br />

10<br />

4<br />

a) Bilde aus den Zahlen 5,6; – 8; – 4,2 und 3,7 alle möglichen verschiedenen Summen mit zwei<br />

Summanden und sortiere die Ergebnisse der Größe nach.<br />

– 12,2 < – 4,3 < – 2,4 < – 0,5 < 1,4 < 9,3<br />

b) Bilde mit jeweils zwei der Zahlen – 2,7; 6,1 und 0,9 alle möglichen Differenzen und sortiere die<br />

Ergebnisse der Größe nach.<br />

– 8,8 < – 5,2 < – 3,6 < 3,6 < 5,2 < 8,8<br />

<br />

Setze die richtigen Zeichen (+ oder –) vor die Zahlen in den Klammern.<br />

a) (j15) – + (j8) + = – 7 b) (j15) – – (j8) + = – 23 c) (j15) – – (j8) – = – 7<br />

(j9) – + (j13 – ) = – 22 (j9) – – (j13) – = + 4 (j13) – + (j9) + = – 4<br />

(j7,5) – + (j8,6) + = + 1,1 (j7,5) – – (j8,6) – = + 1,1 (j8,6) – – (j7,5) + = – 16,1<br />

(j12,8 – ) + (j4,2) – = – 17 (j12,8) – – (j4,2) – = – 8,6 (j12,8) + + (j4,2) – = + 8,6<br />

6


Rationale Zahlen<br />

Multiplikation<br />

1<br />

Berechne im Kopf.<br />

a) (– 8) ∙ 13 = b) ​ 3__<br />

2 ​∙ ( ​ – ​ 1__<br />

3 ​ – ​__<br />

– 104<br />

1 )​= 2 ​ c) (– 0,1) ∙ (– 25) = 2,5 <br />

d) ​( – ​ 1__<br />

5 ​ )​∙ 0,5 = – 0,1<br />

e) (– 0,4) ∙ (– 10) ∙ (– 2,3) = – 9,2 f) ​ 1__<br />

4 ​∙ ( ​ – ​ 1__<br />

2 ​ )​∙ (– 8) = 1<br />

g) (– 1,1) 2 = 1,21<br />

h) – 1,2 2 = – 1,44<br />

i) 0,25 ∙ (– 0,5) ∙ (– 2) = 0,25 <br />

2<br />

Berechne.<br />

a) 0,5 ∙ (– 2)<br />

b) – 3<br />

c) ​ 1__<br />

4 ​ ∙ (– 6)<br />

d) – ​ 2__<br />

∙ (– 6)<br />

⎯⎯→<br />

⎯⎯→<br />

jjjj ∙ ( ​ – ​<br />

– 1<br />

1__<br />

4 ​ )​ __<br />

⎯⎯→<br />

​<br />

jjjj 1 4 ​ ⎯⎯→<br />

jjjj ∙ 0,8 ​__<br />

1<br />

5 ​<br />

∙ ​ 4__<br />

3<br />

⎯⎯→ jjjj ​<br />

⎯⎯→<br />

jjjj ∙ 2,5<br />

∙ ( ​ – ​<br />

– 4<br />

– 10<br />

1__<br />

5 ​ )​<br />

⎯⎯→ jjjj<br />

2<br />

⎯⎯→ – __ ​<br />

jjjj 3 ⎯⎯→<br />

jjjj ∙ 0,1<br />

∙ (– 8)<br />

⎯⎯→<br />

jjjj ∙ ​5__ 3<br />

2 ​<br />

– 0,15<br />

1,2<br />

​<br />

2 ​ )​ – __ ​ ∙ (– 9)<br />

⎯⎯→ jjjj 1 ⎯⎯→<br />

jjjj ∙ ( ​ – ​ 5__<br />

3 ​ )​<br />

⎯⎯→ jjjj ∙ ( ​ – ​ 3__<br />

3 ​ 3<br />

– 5<br />

3 ​ ∙ ​( ​ 1__<br />

∙ (– 3)<br />

⎯⎯→<br />

⎯⎯→<br />

5 ​ )​<br />

⎯⎯→<br />

– 1,2<br />

– 6<br />

2<br />

3<br />

3<br />

Multiplikationsmauern<br />

a) ​ 3 ___<br />

16 ​<br />

– ​ 3 __<br />

4 ​ – ​1 __<br />

4 ​<br />

– ​ 3 __<br />

2 ​ ​1 __<br />

2 ​ – ​1 __<br />

2 ​<br />

__ ​ 3 2 ​ – 1 – ​1 __<br />

2 ​ 1<br />

– 3 – ​ 1__<br />

2 ​ 2 – ​1__<br />

– 3,2 1,28<br />

– 2 1,6 0,8<br />

1 – 2 – 0,8 – 1<br />

– 4 – 0,25 8 – 0,1 10<br />

4 ​ – 4 b) – 4,096<br />

4<br />

5<br />

Tina Klecksel hat Tinte über ihre Matheaufgaben laufen lassen. Ergänze die fehlenden Zahlen.<br />

a) ​( – ​ 3__<br />

4 ​ )​∙ – ___ ​ 16 = 4 b) 0,5 ∙ (– 2,5) = c) ∙ 1,2 = – 6<br />

3 ​<br />

– 1,25<br />

– 5<br />

6<br />

6 ​ )​= – 5 e) (– 0,8) ∙ 0 = 0 f) 1,6 ∙ (– 1,6 ) = – 2,56<br />

d) ∙ ​( – ​ 5__<br />

Multipliziere geschickt.<br />

a) (– 4) ∙ (– 2,9) ∙ (– 2,5) = – 29<br />

b) ​ 3__ – ​__<br />

6<br />

​∙ ​1__ ​∙ (– 16) = 5 ​ 5 8<br />

c) 20 ∙ ​( – ​ 7__<br />

9 ​ ____ ​ 700<br />

)​∙ (– 5) = 9 ​ d) 33 ∙ ​( – ___ ​ 5<br />

12 ​ )​∙ (– 24 ) = 330<br />

e) (– 5) ∙ ​( – ​ 5__ ___<br />

5 ​ )​∙ (– 2) = – 20<br />

f) 0,2 ∙ 0,3 ∙ (– 3) ∙ (– 50) = 9<br />

6 ​ )​∙ ​( ​ 12<br />

6<br />

0,2<br />

0,5<br />

– 1_<br />

2 – 5<br />

Schreibe in die das<br />

Produkt der Zahlen<br />

aus den angrenzenden Feldern.<br />

− 9<br />

1_<br />

3<br />

0,6<br />

− 0,2<br />

– 5_<br />

2<br />

5 16<br />

4<br />

– 0,8<br />

5_<br />

6<br />

– 15 0,6<br />

2<br />

− 1,5<br />

37


Rationale Zahlen<br />

Division<br />

<br />

Berechne im Kopf.<br />

a) 24 :(– 4) = – 6<br />

b) (– 35) :(– 7) = 5<br />

– __ 1<br />

c) (– 1) : 5 = 5<br />

d) (– 5__ – ___<br />

9 ) 1<br />

: 10 = e) (– 3,2) :( –0,8) = f)<br />

1__<br />

18<br />

4<br />

4 : ( – 1__ – __<br />

2 ) 1 = 2<br />

g) 15 :(– 0,5) = h) (– 6) : 3__ = i) 2<br />

1__<br />

– 30<br />

– 10<br />

:(– 0,2) =<br />

– 12 __ 1<br />

5 2 2<br />

– __ 1<br />

k) 1 :(– 4) = 4<br />

l) 0 : (– 1__<br />

2 ) = 0<br />

m) (– 1__<br />

3 ): 1__<br />

8 = – __ 8<br />

3<br />

2<br />

Ergänze.<br />

a)<br />

: – 4 2,5 10<br />

b)<br />

: – 3 0,5 – 1 3__<br />

7<br />

– 10 2,5 – 4 – 1<br />

0 0 0 0 0<br />

1,5 – __ 3<br />

8<br />

– 4,5 1 __ 1<br />

8<br />

0,6 0,15<br />

– 1,8 – 0,45<br />

3__<br />

5<br />

– __ 1<br />

5<br />

– ___ 9<br />

___<br />

10<br />

10 3<br />

1 __ 1<br />

5<br />

– __ 3<br />

5<br />

– 1,8 ___ 9<br />

10<br />

7__<br />

5<br />

– 2,1<br />

<br />

Berechne und bestimme die Summe deiner Lösungen.<br />

a) b)<br />

25 : (– 5)<br />

– 5 2 : ( – 2__<br />

– 3<br />

– 4 2 : ( – 1__<br />

– 6,4<br />

(– 0,8) : (– 2) 0,4 – 1,5 : (– 3) 0,5<br />

– 9<br />

10 : 3 – 0,3 (– 6,8) : 1,7 – 4<br />

0,6 (– 6) : (– 10)<br />

0,2<br />

2)<br />

4)<br />

3)<br />

1__<br />

– 1 4 : (– 1__<br />

0<br />

(– 0,8) : 1__<br />

8<br />

6)<br />

– 1__<br />

6 : (– 5__<br />

0 : 0,7<br />

Summe: – 9,3<br />

Lösung: Die Summe der Summen aus a) und b) ergibt – 22.<br />

– 12,7<br />

4<br />

Berechne. Das Ergebnis der ersten Aufgabe ist die erste Zahl der zweiten Aufgabe usw. Wenn du<br />

nacheinander die zugehörigen Buchstaben notierst, kannst du das Lösungswort erkennen.<br />

(– 6) : 0,5 = – 12 N (– 2__<br />

5) : ( – 4__ ___ 9<br />

= 10 C (– 10) :<br />

9) (– 2__<br />

5) = 25<br />

H 25 : 5__ = 20<br />

E (– 0,25) : 1<br />

1__<br />

4 4 = – __ 1 I (– 3__ :<br />

2) 4__<br />

5 = – ___ 15<br />

5<br />

8<br />

5__<br />

E – 1 7__<br />

8 :(– 3) = 8<br />

R (– 12) : (– 4__ = C<br />

3) 9<br />

2__ :(– 0,4) = – 1<br />

5<br />

E 9 :(– 0,9) = – 10<br />

R – 0,2 : 20 = – 0,01<br />

B 5__ – __ 1<br />

:(– 2,5) = 4<br />

8<br />

2__<br />

S ___ 9<br />

10 : 9__ = 5<br />

H (– 1) :<br />

2__ – __ 3 – __ 2<br />

= 2<br />

E 20 :(– 50) = 5<br />

4 3<br />

Lösungswort:<br />

R ECHENSCHIEBER<br />

8


Rationale Zahlen<br />

Multiplikation und Division<br />

1<br />

2<br />

Ergänze.<br />

a) 1. Faktor – 3 6 ​ 3__<br />

4 ​ – ​2__ 3 ​<br />

2. Faktor 5 1,5 – ​ 4__<br />

5 ​ ​ 1 __<br />

4 ​<br />

Produkt – 15 9 – ​ 3 __<br />

5 ​ – ​ 1__<br />

b) Dividend – 12 9,6 – ​ 4__<br />

3 ​ – 14<br />

Divisor – 4 – 0,8 – 0,5 ​ 7__<br />

8 ​<br />

6 ​ Quotient 3 – 12 2 __ ​ 2 3 ​ – 16<br />

Berechne die Produkte und Quotienten. Die richtigen Ergebnisse findest du im Zahlenfeld. Färbe sie<br />

blau.<br />

– 4 :(– 8) = 0,5<br />

0,91 ∙ (– 10) = – 9,1<br />

2,75 ∙ (– 5,5 ) = – 15,125<br />

– ​ 2__<br />

3 ​∙ ( ​ – ​ 3__<br />

8 ​ ​__<br />

1<br />

)​= 4 ​<br />

​ 3__ 5 ​: ( ​ – ​ 3__<br />

8 ​ – __ ​ 8 )​= 5 ​<br />

– 1,5 : 0,2 = – 7,5<br />

2,2 ∙ (– 0,8) = – 1,76<br />

1​ 1__ ​: 5 = 0,3<br />

(– 1,44 ):(– 1,2 ) = 1,2<br />

2<br />

– 17 ∙ (– 0,4) = 6,8<br />

(– 1,5) 2 = 2,25<br />

(– 2,5) ∙ ​( – ​ 3__<br />

2 ​ 3 __ ​ 3 )​= 4 ​<br />

– ​ 5__ – ​__<br />

5<br />

​: ​3__ ​= 3 ​<br />

15,12 :(– 2,7) = – 5,6<br />

​8<br />

9 9 ___<br />

15 ​: ( ​ – ​ 4__<br />

5 ​ – ​__<br />

2<br />

)​= 3 ​<br />

1,5 ∙ (– 2) ∙ ​( – ​ 3__<br />

2 ​ 7__<br />

)​= 4,5<br />

9 ​: ( ​ – ​ 7__<br />

9 ​ )​= – 1<br />

– 1,8 2 = – 3,24<br />

25 ∙ ​(<br />

– ​__<br />

5<br />

0,4 :(– 8) )​= 4 ​<br />

– ​ 2__ ​ 1,6 – 1,76 5,6 – 5,6 10 6,8 9,1 1,2 2,25 ​1__<br />

3 2 ​<br />

– 15,125 – 1,25 4 ​ 1__ ​ 1 – 7,5 ​1__<br />

2 5 ​ 3 ​3__ ​ 3,24 – 9,3 – 3,24 – 2<br />

4<br />

– 9,1 – 2 ​ 1__<br />

4 ​ – 1​2__ 3 ​ ​2__ 3 ​ – 1 ​1__ ​ 0,3 – 10 – ​5__<br />

4 6 ​ – 1,6 0<br />

3<br />

Finde die Zahl für x und mache die Probe.<br />

a) – 5 ∙ x = 7,5 x = – 1,5<br />

Probe: – 5 ·(– 1,5) = 7,5<br />

b) x ∙ (– 3) ∙ 4 = – 48 x = 4<br />

Probe: 4 ·(– 3) · 4 = – 48<br />

– ​__<br />

1<br />

c) (– 1,5) : x = 3 x = 2 ​<br />

Probe: – 1,5 : ​( – ​__<br />

1<br />

2 ​ )​ = 3<br />

d) ​ 3__ ​∙ ​2__ ​∙ x = – 4 x = – 8<br />

​__<br />

Probe: <br />

1 ​· (– 8) = – 4<br />

4 3<br />

2<br />

e) ​( – ​ 1__<br />

3 ​ )​ 2 – ​__<br />

1<br />

: x = – 1 x = 9 ​<br />

​__<br />

Probe: <br />

1 9 ​ : ( ​ – ​__<br />

1<br />

9 ​ )​ = – 1<br />

f) – 3 ∙ x = 6 ∙ x x = 0<br />

Probe: – 3 · 0 = 6 · 0<br />

4<br />

Schreibe als Gleichung und bestimme x.<br />

2<br />

a) Wenn du die Zahl x mit (– 4) multiplizierst, erhältst du 8. x ·(– 4) = 8<br />

x = – <br />

e) Das Produkt von (– 5) und x ist ​ 1__<br />

2 ​. (– 5) · x = __ ​ 1 2 ​ x = – ​___<br />

1<br />

10 ​<br />

b) Wenn du 1,6 durch die Zahl x dividierst, erhältst du – 0,2. 1,6 : x = – 0,2 x = – 8<br />

c) Wenn du x mit ​ 3__<br />

x · __ ​ 3 ​multiplizierst, erhältst du – 2. 8 ​ = – 2 8<br />

– ___ ​<br />

x = <br />

3 ​ = – 5 __ ​1 3 ​<br />

d) Das Doppelte der Zahl x ist genauso groß wie ihr Dreifaches. 2 · x = 3 · x x = 0<br />

39


Rationale Zahlen<br />

Vermischtes 1<br />

<br />

2<br />

Berechne.<br />

a) – 3__<br />

8 + 7__ 1 __ 3<br />

= b)<br />

1__<br />

4 5 – ___ 7<br />

10 = – __ 1<br />

8<br />

2<br />

c) (– 5__ ∙<br />

6) (– ___<br />

20) 3 1__<br />

= d) 7__<br />

9 : (– 2__ – 1 __<br />

=<br />

3) 1 8<br />

6<br />

e) 1,24 – (– 0,84) = 2,08<br />

f) 3,5 + (– 7,8) = – 4,3<br />

g) – 0,8 ∙ 16 = – 12,8<br />

h) (– 9,1) :(– 13) = 0,7<br />

i) – 4__<br />

5 ∙ 0,6 = – 0,48<br />

k) – 0,7 + (– 3 1__ =<br />

2) – 4,2<br />

Lösungen: – 12,8; – 4,3; – 4,2; – 1 1__<br />

6 ; – 1__ ; – 0,48;<br />

1__ 3__<br />

; 0,7; 1<br />

2 8 8 ; 2,08<br />

Finde heraus, welche Aufgaben falsch gerechnet sind. Korrigiere sie.<br />

Die Buchstaben, die bei den falsch gerechneten Aufgaben stehen, führen dich<br />

zum Lösungswort.<br />

0,2 – 0,8 + 2,6 = 2<br />

1__<br />

4 – 1__<br />

3 – 1__<br />

2 = – ___ 7<br />

3__<br />

12<br />

8 – 1__<br />

8 ∙ 4 = 1 – __<br />

R<br />

E<br />

1 8 O<br />

2,5 :(– 0,5) – (– 1) = – 4<br />

G<br />

– 1,7 2 = 2,89<br />

– 2,89 N<br />

– 2 + 2 ∙ (– 1,5 ) = – 5<br />

E<br />

1 – (– 1__<br />

2 – 2 ) = – 1 1__ 0,9 :(– 0,3) = – 3<br />

2<br />

(– 2__<br />

3 ∙ 3__<br />

4) :(– 2) = 1<br />

1__<br />

3,5 I<br />

S<br />

4 M<br />

2,5 :(– 3__<br />

8 – 1__ = – 10<br />

8) – 5__<br />

6 + (– 2__ ∙ 4 = – 3,5<br />

3) 1,8 ∙ (– 0,3) ∙ 0 = – 0,54<br />

– 5 O<br />

N 0 D<br />

Lösungswort: DOMINO<br />

Korrigierte Ergebnisse:<br />

– 5; – 2,89; – 1__<br />

8 , 0;<br />

1__<br />

4 ; 3,5<br />

<br />

Berechne. Immer zwei Aufgaben haben das gleiche Ergebnis. Gib die Paare an.<br />

a) – 5,2 + 1,4 ∙ 3 = – 1<br />

b) 2,3 – (– 0,5 – 2 ) = 4,8<br />

c) (– 1,5) 2 :(– 5 ) = – 0,45<br />

d) 1 :(– 2) :(3 – 4) = 0,5<br />

e) (– 1,8) ∙ (– 0,2 ) + (– 1,66) = – 1,3<br />

f) 4,8 : (– 1__ :(– 2) =<br />

2) 4,8<br />

g) 2,8 :(– 0,7) ∙ (– 1__ =<br />

8) 0,5<br />

h) 1__<br />

2 – 4__<br />

5 : 8<br />

15 = – 1<br />

i) 1 1__<br />

2 + (– 2 1__ ___ =<br />

10<br />

– 0,45<br />

k) 16,9 :(– 1,3) : 10 = – 1,3<br />

4) + 3<br />

Lösungspaare:<br />

a) b) c) d) e)<br />

h) f) i) g) k)<br />

4<br />

In einer Skatrunde macht Martin bei einem Punktestand von – 94 das Spiel.<br />

Gewinnt er, bekommt er 72 „Pluspunkte“, verliert er, gibt es 144 „Minuspunkte“.<br />

Wie könnte Martins Punktestand nach dem Spiel aussehen?<br />

Bei Sieg: – 22<br />

Bei Niederlage: – 238<br />

40


Rationale Zahlen<br />

Vermischtes 2<br />

1<br />

Notiere zunächst einen Term und berechne dann.<br />

a) Welchen Abstand haben die Zahlen – 4 ​ 3__ ​und 2 ​1__ ​auf der Zahlengeraden?<br />

4 4<br />

​| – 4 ​ 3 __<br />

4 ​ |​ + 2 ​ 1 __<br />

4 ​ = 7<br />

b) Welche Zahl liegt auf der Zahlengeraden in der Mitte zwischen – 3,7 und – 15,2?<br />

– 9,45<br />

c) Welche negative Zahl ist von – 2,4 doppelt so weit entfernt wie von 1?<br />

– ___ ​ 2<br />

2<br />

15 ​ 2,23<br />

Merle und Frank haben zwei ganz besondere Spielwürfel:<br />

Einen weißen mit 12 Flächen (Dodekaeder) mit den Zahlen von<br />

1 bis 12 und einen grünen mit 20 Flächen (Ikosaeder) mit den<br />

Zahlen von 1 bis 20.<br />

Sie vereinbaren, dass der weiße Würfel immer positive Zahlen liefert<br />

und der grüne Würfel negative Zahlen. Beide Würfel werden<br />

gleichzeitig geworfen.<br />

Bestimme die kleinst- bzw. größtmöglichen Werte für Summe,<br />

Differenz, Produkt und Quotient der beiden gewürfelten Zahlen.<br />

10<br />

8<br />

3<br />

7<br />

4<br />

12<br />

15<br />

8<br />

12<br />

13<br />

20<br />

7<br />

14<br />

11<br />

9<br />

Summe Differenz Produkt Quotient<br />

kleinstmöglicher Wert – 20 + 1 = – 19 – 20 – 12 = – 32 12 ·(– 20) = – 240 (– 20) : 1 = – 20<br />

größtmöglicher Wert – 1 + 12 = 11 12 – (– 20) = 32 1 ·(– 1) = – 1 1 :(– 20) = – ​___<br />

1<br />

20 ​<br />

3<br />

Fülle die Lücken in der Rechenkette aus.<br />

a) +<br />

·<br />

: –<br />

:<br />

·<br />

1__<br />

− 2,5 – 4<br />

11<br />

3 –<br />

__<br />

4,5<br />

2__<br />

2<br />

– − 5 − 0,4<br />

3<br />

– 2,75 – 8 ​__<br />

1<br />

4 ​ 1 __ ​1 2 ​ – 3 2<br />

b)<br />

: +<br />

: – :<br />

·<br />

–<br />

5__<br />

6<br />

5__<br />

3<br />

– ​__<br />

1 ​ 0,9 – – 1,62 4 ___ ​1<br />

2 20 ​ 4 ​1 __<br />

2 ​<br />

1,4 1,8 ___ 9<br />

– 0,4<br />

1__<br />

– − 9 −<br />

20<br />

2<br />

4<br />

Berechne mit dem Taschenrechner.<br />

a) ​ 7 – 0,5 ∙ ( ​ ​ 3__<br />

______________ 4 ​+ 2,1 )​<br />

3 ​ 1__ ​– ​3__<br />

4<br />

c) __________________<br />

– 4 ∙ (– 0,2 – 5,9) + 12,4<br />

​<br />

​ 2__<br />

3 ​: ( ​ ​ 1__<br />

6 ​– 2 )​<br />

4 ​ ​= jjj b) ​<br />

​= – jjj 101,2<br />

d) ​ ​2__ 5<br />

________________<br />

2,5 : 1,5 ∙ (– 1,8 + 7,8)<br />

– ​ 3__ ​+ 7 ∙ ​3<br />

5 ___<br />

​– (– 0,6) + ​3__<br />

___________ 8 ​<br />

8 – ​( – ​ 3__ ​+ 6 ​3__<br />

2 4 ​ )​<br />

​= jjj 6 __ ​ 2<br />

10 ​ 3 ​<br />

​= jjj 0,5<br />

Lösungen: 0,5; 2,23; 6 ​ 2__ ​; – 101,2<br />

3<br />

41


Rationale Zahlen<br />

Terme mit rationalen Zahlen<br />

<br />

Stelle einen Term auf und berechne wie im Beispiel.<br />

Multipliziere die Differenz von<br />

– 7 und – 8,5 mit 1,6.<br />

(– 7 – (– 8,5)) ∙ 1,6 = 1,5 ∙ 1,6<br />

= 2,4<br />

(– 3,4 : 17) + 3 __ 1<br />

2<br />

– __<br />

=<br />

5 + 3 __ 1<br />

2<br />

a) Addiere 3 1__ zu dem<br />

2<br />

Quotienten aus – 3,4 und<br />

17.<br />

= 3,3<br />

b) Subtrahiere von – 12 die<br />

Summe aus – 4,06 und<br />

– 8,4.<br />

c) Multipliziere die Differenz<br />

=<br />

von – 2 1__ und 1<br />

1__ 6__<br />

mit –<br />

3 2 5 . =<br />

d) Dividiere das Produkt<br />

von 1,5 und – 8 durch<br />

den Quotienten von –1<br />

und– 0,6.<br />

f) Subtrahiere den<br />

Quotienten aus – 3,24 und<br />

0,4 von der Summe der<br />

Zahlen – 2,1 und – 5,94.<br />

– 12 – ( – 4,06 + (– 8,4) ) – 12 + 12,46<br />

=<br />

= 0,46<br />

( – 2 1 __<br />

3 – 1 1 __<br />

2 ) · ( – 6 __<br />

5 ) – 3 5 __<br />

6 · ( – 6 __<br />

( 1,5 · (– 8) ) : ( – 1 : (– 0,6) )<br />

=<br />

= – 7,2<br />

e) Addiere die Differenz von<br />

=<br />

– 2__ 7__<br />

und – zu dem Produkt<br />

3 9<br />

aus 3__ 8__<br />

und –<br />

4 9 . – __<br />

=<br />

5 9<br />

(– 2,1 + (– 5,94) ) – (– 3,24 : 0,4) = – 8,04 – (– 8,1)<br />

4 3 __<br />

5<br />

– 12 : 1 2 __<br />

3<br />

3__<br />

4 · (– __ 8<br />

9 ) + (– __ 2<br />

3 – (– __ 7<br />

9 ) ) – __ 2<br />

3 + __ 1<br />

9<br />

= 0,06<br />

5 )<br />

Lösungen: – 7,2; – 5__<br />

3__<br />

; 0,06; 0,46; 3,3; 4<br />

9 5<br />

2<br />

Setze Klammern so, dass das Ergebnis stimmt.<br />

( 20 – 35 ) ∙ 7 = – 105 2 – 1__<br />

2 ∙ ( 3__<br />

4 – 1 ) = 2 1__<br />

8<br />

– 0,5 + ( 3,7 – 1,2 ) : 5 = 0 (– 5__<br />

6 + 1__<br />

3)<br />

∙ ( – 2__ –<br />

3) 1__<br />

6 ) = ___ 5<br />

12<br />

(– 7 – 8 ): 5 + 3 ∙ (– 1__<br />

( )<br />

2 – 4 = – 72<br />

2) = – 4,5 – 2 ∙ 9 2 : (– 3) : 5 1__<br />

( )<br />

a) Während einer Skifahrt hat Klaus jeden Morgen zur gleichen Zeit die<br />

Außentemperatur gemessen und in nebenstehender Liste notiert.<br />

Wie hoch war die durchschnittliche Morgentemperatur in dieser Woche?<br />

– 2,9 °C<br />

3.1. 4.1. 5.1. – 6° C<br />

– 8,3° C<br />

– 1,8° C<br />

6.1. 1,3° C<br />

b) Hätte Klaus auch am 10.1. die Temperatur gemessen, hätte sich die<br />

Durchschnittstemperatur nicht geändert. Wie kalt war es am 10.1?<br />

7.1. 8.1. 1,6° C<br />

– 2,7° C<br />

– 2,9 °C<br />

9.1. – 4,4° C<br />

42


Gleichungen und Terme<br />

Gleichungen aufstellen und lösen 1<br />

1<br />

Stelle eine Gleichung auf und löse sie durch Probieren.<br />

a) Von zwei Zahlen ist eine um 8 größer als die andere. Ihre Summe beträgt 42.<br />

x + (x + 8) = 42 x = 17<br />

b) Ein Rechteck ist eineinhalb mal so lang wie breit. Sein Umfang beträgt 40 m.<br />

2 · ​( x + ​ 3 __<br />

<br />

2 ​ x )​ = 40<br />

x = 8; Breite: 8 cm, Länge: 12 cm<br />

2<br />

Löse mithilfe der Tabelle.<br />

Leo und sein Vater haben am gleichen Tag<br />

Geburtstag. Als Leo 11 wird, wird sein Vater 35.<br />

Vor wie vielen Jahren war Leos Vater viermal so<br />

alt wie sein Sohn?<br />

Anzahl der Jahre<br />

x<br />

Alter von Leo<br />

11 – x<br />

Alter des Vaters<br />

35 – x<br />

1 10 34<br />

2 9 33<br />

3 8 32<br />

fehlt hier 1 Zeile?<br />

3<br />

Löse mithilfe der Waage.<br />

Entferne dazu so weit wie möglich alle Gegenstände, die auf beiden Seiten der Waage liegen.<br />

a) Wie schwer ist eine Vase? b) Wie schwer ist ein Ball?<br />

1 Vase 1 Kugel 4 Gewichte 6 Kugeln<br />

Eine Kugel ist 200 g schwer.<br />

Ein Gewicht ist 600 g schwer.<br />

Eine Vase wiegt 200 g.<br />

Ein Ball wiegt 400 g.<br />

c) Wie schwer ist ein Hut? d) Wie schwer ist eine Katze?<br />

2 Hüte 4 Schals 1 Katze<br />

2 Steine<br />

Ein Schal wiegt 150 g.<br />

Ein Stein wiegt 1 ​ 1__<br />

2 ​kg.<br />

Ein Hut wiegt 300 g.<br />

Eine Katze wiegt 3<br />

kg.<br />

43


Gleichungen und Terme<br />

Gleichungen aufstellen und lösen 2<br />

<br />

Stelle eine Gleichung auf und gib die Lösung an.<br />

a) Das Doppelte einer Zahl ist – 78. 2 · x = – 78<br />

1__<br />

3 · x = __ 1<br />

4<br />

b) Der dritte Teil einer Zahl ist 1__<br />

4 .<br />

x = 3 __<br />

4<br />

x = – 39<br />

c) Der Vorgänger einer natürlichen Zahl ist 34. x – 1 = 34 x = 35<br />

1__ ·(x + 3) = 15 x = 27<br />

d) Die Hälfte der um 3 vergrößerten Zahl ist 15. 2<br />

x + __ 1 x = 18 x = 12<br />

e) Die Summe aus einer Zahl und der Hälfte dieser Zahl ist 18. 2<br />

f) Das Dreifache einer Zahl, vermindert um 5, ergibt 1. 3 x – 5 = 1 x = 2<br />

2<br />

Welche Geschichte passt zu welcher Gleichung?<br />

Susi kauft für ihre Mutter Blumen.<br />

Eine Rose kostet 1,50 €.<br />

Weil die Verkäuferin ihr eine<br />

Freude machen will, schenkt sie<br />

ihr noch eine Rose. Susi muss 9 €<br />

bezahlen.<br />

D<br />

Sven kauft sich Birnensaft, von dem<br />

jede Packung 1,50 € kostet, und eine<br />

Sportzeitschrift für 1 €.<br />

An der Kasse bezahlt er 10 €.<br />

F<br />

Herr Frey kauft sich<br />

in seiner Mittagspause<br />

ein belegtes<br />

Brötchen für 1,50 €<br />

und für den Nachmittag<br />

Schokolade<br />

– die Tafel zu<br />

0,50 €. Er muss 9 €<br />

bezahlen.<br />

C<br />

A 1,5 + x + 1 = 10<br />

B 1,5 ∙ x + 1 = 9<br />

C 1,5 + 0,5 ∙ x = 9<br />

D 1,5 ∙ (x – 1) = 9<br />

E 1,5 ∙ x + 0,5 = 9 F 1,5 ∙ x + 1 = 10<br />

G (1,5 + 0,5) ∙ x = 9<br />

<br />

Stelle eine Gleichung auf und löse sie mit einer Methode deiner Wahl (Probieren, Tabelle, Waage).<br />

a) Beim Handballturnier der 7 b haben Thorben und Mareike zusammen<br />

35 Tore geworfen. Hätte Thorben 4 Treffer mehr und Mareike einen<br />

Treffer weniger erzielt, dann wären beide gleich oft erfolgreich gewesen.<br />

Wie viele Tore hat jeder der beiden erzielt?<br />

Anzahl der Tore von Thorben: x<br />

Anzahl der Tore von Mareike: jjj 35 – x Gleichung: x + 4 = 35 – x – 1<br />

Thorben: 15 Mareike: 20<br />

b) Carla bekommt dreimal so viel Taschengeld wie ihr Bruder Felix.<br />

Nach einer Taschengelderhöhung von 5 € für jedes Kind hat Carla nur noch<br />

doppelt so viel Taschengeld wie Felix.<br />

Wie hoch war das Taschengeld von Carla und Felix vor der Erhöhung?<br />

Höhe von Felix Taschengeld<br />

Höhe von Carlas Taschengeld<br />

vor der Erhöhung: jjj x<br />

vor der Erhöhung: jjj 3 · x<br />

nach der Erhöhung: jjj x + 5<br />

nach der Erhöhung: 3 jjj · x + 5<br />

Gleichung: (x + 5) · 2 = 3 · x + 5<br />

2 x + 10 = 3 x + 5 x = 5<br />

44


Gleichungen und Terme<br />

Gleichungen umformen 1<br />

1<br />

Gib jeweils die Umformung an, die die erste Gleichung in die zweite Gleichung überführt.<br />

jjj – 5<br />

jjj : 3<br />

a) x + 5 = 17 ⎯⎯→ x = 12 b) 3 ∙ x = 25,5 ⎯⎯→ x = 8,5<br />

jjj · 4<br />

jjj – x<br />

c) ​ 1__<br />

4 ​ x = – 8 ⎯⎯→ x = – 32<br />

d) 2 x = x – 7,6 ⎯⎯→ x = – 7,6<br />

jjj + 10<br />

jjj : 3<br />

e) 3 x – 10 = – 7 ⎯⎯→ 3 x = 3 f) 3 x = 3 ⎯⎯→ x = 1<br />

2<br />

Löse die Gleichungen mithilfe der angegebenen Umformungen.<br />

Überprüfe deine Lösung, indem du sie in der Ausgangsgleichung einsetzt.<br />

+ 6<br />

: 2<br />

a) 2 x – 6 = 3 ⎯⎯→ jjjjjj 2 x = 9 ⎯⎯→ x = jjj 4,5<br />

b) ​ 3__<br />

4 jjjjjj ​x + 8 = 7 ⎯⎯→ – 8 __<br />

: ​<br />

​ 3 3__<br />

4 ​<br />

4 ​ x = – 1 ⎯⎯→ x = jjj – __ ​ 4 3 ​<br />

+ x<br />

: 5<br />

c) 5 – x = 4 x ⎯⎯→ jjjjjj 5 = 5 x ⎯⎯→ x = jjj 1<br />

– 3<br />

: 1,5<br />

⎯⎯→ 1,5 x = 9 ⎯⎯→ 6<br />

d) 3 + 1,5 x = 12 jjjjjj x = jjj<br />

3<br />

Löse die Gleichungen wie im Beispiel.<br />

Überprüfe deine Lösung, indem du sie in der Ausgangsgleichung einsetzt.<br />

5 x + __ ​ 3 4 ​ = __ ​1 ​ + 2 x<br />

2<br />

| – 2 x a) 6 x + 2,5 = 2 x – 1,5 | – 2 x<br />

3 x + __ ​ 3 4 ​ = __ ​1 2 ​ | – __ ​3 4 ​ 4 x + 2,5 = – 1,5<br />

| – 2,5<br />

3 x = – __ ​ 1 4 ​ | : 3 4 x = – 4<br />

| : 4<br />

x = – ___ ​ 1<br />

12 ​ x = – 1<br />

Probe: 5 ∙ ​( – ​___<br />

1<br />

12 ​ )​ + __ ​ 3 4 ​ = __ ​1 2 ​ + 2 ∙ ( ​ – ​___<br />

1<br />

12 ​ )​ Probe: 6 ·(– 1) + 2,5 = 2 ·(– 1) – 1,5<br />

__ ​ 1 3 ​ = __ ​1 3 ​ <br />

– 3,5 = – 3,5<br />

b) 3 – 2 x = 8 – 7 x | + 7 x c) – 4 + 4 x = – 1 – ​ 4__<br />

5 ​x | + ​__<br />

4<br />

5 ​ x<br />

3 + 5 x = 8<br />

| – 3<br />

– 4 + 4 ​__<br />

4 = – 1<br />

5 ​ x<br />

| + 4<br />

5 x = 5<br />

| : 5<br />

4 ​__<br />

4 = 3<br />

| : ___ ​ 24<br />

5 ​ x 5 ​<br />

x = 1<br />

x = ___ ​ 15<br />

24 ​ = __ ​5 8 ​<br />

Probe: 3 – 2 · 1 = 8 – 7 · 1<br />

Probe: – 4 + 4 · ​__<br />

5<br />

8 ​ = – 1 – ​4 __<br />

5 ​· ​5 __<br />

8 ​<br />

d) – ​ 3__ ​+ 4 x = – 1,5 + x | – x<br />

e) 7 x – 3 = 4 – 7 x | + 7 x<br />

2<br />

– __ ​ 3 = – 1,5<br />

| – ​__<br />

3 = |<br />

2 ​ + 3 x<br />

2 ​<br />

14 x – 3 4<br />

+ 3<br />

Probe:<br />

1 = 1<br />

<br />

3 x = 0<br />

| : 3<br />

14 x = 7<br />

| : 14<br />

x = 0<br />

x =<br />

– ​__<br />

3<br />

2 ​ + 4 · 0 = – 1,5 + 0<br />

Probe: <br />

– 1,5 = – 1,5 <br />

<br />

– 1,5 = – 1,5 <br />

​ 1 __<br />

2 ​<br />

7 · ​ 1 __<br />

2 ​ – 3 = 4 – 7 · ​1 __<br />

2 ​<br />

0,5 = 0,5<br />

<br />

45


Gleichungen und Terme<br />

Gleichungen umformen 2<br />

<br />

Löse die Gleichungen mithilfe der angegebenen Umformungen auf zwei verschiedene Arten.<br />

a) 2 x – 8 = 4<br />

+ 8<br />

⎯⎯→<br />

: 2<br />

2 x = 12<br />

⎯⎯→<br />

2 x – 8 = 4<br />

: 2<br />

⎯⎯→<br />

x – 4 = 2<br />

+ 4<br />

⎯⎯→<br />

b) 5 – x = – 8<br />

+ x<br />

=<br />

⎯⎯→<br />

5 – 8 + x<br />

+ 8<br />

⎯⎯→<br />

5 – x = – 8<br />

+ 8<br />

⎯⎯→<br />

13 – x = 0<br />

+ x<br />

⎯⎯→<br />

– 7<br />

c) 4 x + 7 = – 3 ⎯⎯→<br />

4 x = – 10<br />

: 4<br />

⎯⎯→<br />

: 4<br />

x + __ 7 – __ 3<br />

4 x + 7 = – 3 ⎯⎯→<br />

4 = 4<br />

jjj<br />

– __ 7<br />

4<br />

⎯⎯→<br />

1__<br />

d) 6 + 1__<br />

2 x = 4 – 6<br />

=<br />

jjj<br />

2 x – 2<br />

· 2<br />

jjj<br />

⎯⎯→<br />

6 + 1__<br />

2 x = 4 ∙ 2<br />

12 + x = 8<br />

– 12<br />

⎯⎯→<br />

x = jjj 6<br />

x = jjj 6<br />

x = jjj 13<br />

x = jjj 13<br />

x = jjj – 2,5<br />

x = jjj – 2,5<br />

x = jjj – 4<br />

x = jjj – 4<br />

2<br />

<br />

Durch die angegebenen Umformungen ist die einfache Gleichung entstanden.<br />

Wie sah die Ausgangsgleichung aus?<br />

+ 6<br />

: 3<br />

a) 3 jjj x – 6 = jjj 6 ⎯⎯→ jjj 3 x = jjj 12 ⎯⎯→ x = 4<br />

b) 2 jjj x + 10 = jjj 7 – 10<br />

⎯⎯→ jjj 2 x = jjj – 3 : 2<br />

⎯⎯→ x = – 1,5<br />

3__<br />

1__<br />

c) jjj =<br />

– 2 3__<br />

:<br />

jjj ⎯⎯→ jjj = jjj 3__<br />

4 x = – 2<br />

2<br />

4 x – __ 3<br />

4 x – 2<br />

2 ⎯⎯→<br />

– 2 x<br />

: 5<br />

d) jjj 7 x = 10 jjj + 2 x ⎯⎯→ jjj 5 x = jjj 10<br />

⎯⎯→ x = 2<br />

Wo steckt der Fehler? Markiere ihn und löse die Gleichung dann richtig.<br />

a) 5 x + 14 = 10 | : 5 b) x + 3 = 1 – x | + x c) 4 – 1__<br />

2 x = 8 | : 1__<br />

2<br />

x + 14 = 2 | – 14 2 x + 3 = 1 | – 3 2 – x = 4 | – 2<br />

x = – 12 2 x = – 2 | : 2 – x = 2 | ∙ (– 1)<br />

x = 0 x = – 2<br />

5 x + 14 = 10 | – 14<br />

5 x = – 4 | : 5<br />

x = – __ 4<br />

5<br />

2 x = – 2 | : 2<br />

x = – 1<br />

4 – __ 1<br />

2 x = 8 | – 4<br />

– __ 1 x = 4<br />

2<br />

| ·(– 2)<br />

x = – 8<br />

46


Gleichungen und Terme<br />

Gleichungen umformen und lösen<br />

1<br />

2<br />

Löse die Gleichungen mithilfe von Umformungen.<br />

a) 5 x + 8 = 3 x – 0,5 | – 3 x<br />

2 x + 8 = – 0,5 | – 2<br />

<br />

<br />

c) 3 x – ​ 1__ ​+ ​2__ ​x = – 4 – 0,6 x<br />

2 5<br />

3 ​__<br />

2<br />

5 ​ x – __ ​1 2 ​ = – 4 – 0,6 x | + ​1 __<br />

2 ​<br />

3,4 x = – 3,5 – 0,6 x | + 0,6 x<br />

<br />

<br />

2 x = – 8,5 | : 2<br />

x = – 4,25<br />

Lösungen: – 5 ​ 3__ ​; – 4 ​1__ ​; – 2 ​1__ ​; – ​7__<br />

5 4 9 8 ​<br />

b) – 2 x – 8 = 2 x + ​ 4__<br />

9 ​ | + 2 x<br />

– 8 = 4 x + __ ​ 4 9 ​ | – ​4 __<br />

9 ​<br />

– 8 __ ​ 4 9 ​ = 4 x | : 4<br />

– 2 ​__<br />

1<br />

9 ​ = x<br />

d) 0,3 x – 0,8 = 1,4 x + 2 – 0,6 x<br />

0,3 x – 0,8 = 0,8 x + 2 | + 0,8<br />

8 ​ x = – 5,6<br />

<br />

<br />

0,3 x = 0,8 x + 2,8<br />

– 0,5 x = 2,8<br />

| – 0,8 x<br />

| :(– 0,5)<br />

4 x = – 3,5 | : 4<br />

x = – ​ 7 __<br />

Löse die Zahlenrätsel mithilfe von Gleichungen.<br />

a) Die Summe von drei Zahlen ist 201. Dabei ist die zweite Zahl um 6 größer als die erste, die dritte<br />

um 6 größer als die zweite.<br />

Gleichung: x + (x + 6) + (x + 12) = 201 Die gesuchten Zahlen sind 61; 67; 73<br />

b) Zwei Zahlen unterscheiden sich um 14. Ihre Summe ist 4.<br />

Gleichung: x + (x + 14) = 4 Die gesuchten Zahlen sind – 5; 9<br />

c) Subtrahiert man vom Fünffachen einer Zahl 7, erhält man das Doppelte der Zahl.<br />

Gleichung: 5 x – 7 = 2 x<br />

Die gesuchte Zahl ist <br />

​__<br />

7<br />

3 ​<br />

d) Dividiert man eine Zahl durch 4 und addiert dann ​ 3__ ​, erhält man die um 2 vergrößerte Zahl.<br />

4<br />

Gleichung: <br />

x : 4 + ​__<br />

3<br />

4 ​ = x + 2 Die gesuchte Zahl ist <br />

– __ ​5 3 ​<br />

3<br />

Löse mithilfe geeigneter Gleichungen.<br />

a) Bestimme die Seitenlängen der Dreiecke.<br />

x + 1<br />

4<br />

x + 1<br />

I<br />

U = 18 cm<br />

x<br />

2 x<br />

II<br />

U = 25 cm<br />

x + 3<br />

I<br />

x + 1 + x + 1 + 4 = 18<br />

<br />

II <br />

b) Von zwei Nebenwinkeln ist der eine viermal so groß wie der andere. <br />

<br />

<br />

2 x = 12 x = 6<br />

x + 2 x + x + 3 = 25<br />

4 x + 3 = 25<br />

x = 5,5<br />

α + 4 · 2 = 180° α = 36° und = 144°<br />

47


Gleichungen und Terme<br />

Gleichungen lösen<br />

Gib bei allen Aufgaben an, welche Größe gesucht ist, stelle damit eine geeignete Gleichung auf und<br />

löse sie.<br />

<br />

a) Zwei Bücher kosten zusammen 42 €. Das eine ist um 4 € teurer als das andere.<br />

Wie viel kosten die Bücher? x: Kosten des 1. Buchs<br />

Gleichung: x + x + 4 = 42 Die Bücher kosten 19 € und 23 €<br />

b) Britta liest in den Ferien drei Bücher mit insgesamt 676 Seiten.<br />

Das zweite Buch hat doppelt so viele Seiten wie das erste, das dritte<br />

dafür 20 Seiten weniger als das erste.<br />

Wie viele Seiten hat jedes Buch? x: Seitenzahl des 1. Buchs<br />

.<br />

Gleichung: x + 2 x + x – 20 = 676 Die Bücher haben 174; 348; 154 Seiten.<br />

2<br />

a) Eine 3 m lange Holzleiste wird in vier Stücke geschnitten. Jedes Stück ist 20 cm länger als das<br />

vorherige. Wie lang sind die Stücke? x: Länge des 1. Stücks<br />

Gleichung: x + (x + 20) + (x + 40) + (x + 60) Die Stücke sind 45 cm; 65 cm; 85 cm; 105 cm.<br />

= 300<br />

b) Eine andere, ebenfalls 3 m lange Leiste, wird so in vier Stücke geschnitten, dass jedes Teil doppelt<br />

so groß ist wie das vorherige. Wie lang sind jetzt die Stücke? x: Länge des 1. Stücks<br />

Gleichung: x + 2 x + 4 x + 8 x = 300 Die Stücke sind 20 cm; 40 cm; 80 cm; 160 cm.<br />

<br />

Im Stadion des FC Schuss wird die neue Tribüne in drei Bereiche aufgeteilt. In den Kurven (K) sitzen<br />

halb so viele Zuschauer wie auf den Geraden (G). Im extra angelegten Familienblock (F) finden 10 %<br />

der Zuschauer Platz. Insgesamt fasst das Stadion 18 000 Zuschauer.<br />

x: Anzahl in G<br />

1__<br />

Gleichung: x + x + 1800 = 18000<br />

2<br />

In K sitzen 5400 , in G 10800 und in F 1800<br />

Zuschauer.<br />

4<br />

Herr Rukel hat im letzten Jahr 2600 € für 5 % angelegt. Mittlerweile ist der Zinssatz leider gefallen.<br />

Wie viel Geld muss Herr Rukel anlegen, um bei einem Zinssatz von 4 % genauso viele Zinsen pro<br />

Jahr zu bekommen?<br />

x: Kapital ____ 4<br />

5 % von 2600 = 130<br />

100 · x = 130<br />

x = 3250 Er muss 3250 € anlegen.<br />

<br />

Herr Steffen ist heute viermal so alt wie seine Tochter Franzi.<br />

In fünf Jahren wird er nur noch dreimal so alt sein wie sie.<br />

Wie alt sind die beiden heute?<br />

x: Alter von Franzi heute<br />

(x + 5) · 3 = 4 x + 5 x = 10<br />

40<br />

10<br />

48


Gleichungen und Terme<br />

Rechnen mit Termen – Einsetzen<br />

1<br />

Berechne den Wert des Terms.<br />

x y 3 – 2 x y (1 – y) + 7 x + 2 y – ​ 1__ ​x ∙ 2 y 5 – 3 ∙ (x + 1) 0,2 x – y ∙ 0,5<br />

2<br />

3 – 2 – 3 1 – 1 6 – 7 1,6<br />

0 ​ 1__<br />

4 ​ 3 7 ​___<br />

3<br />

16 ​ __ ​1 2 ​ 0 2 – __ ​1 8 ​<br />

– 1 1 5 7 1 1 5 – 0,7<br />

– 1,5 – 4 6 – 13 – 9,5 – 6 6,5 1,7<br />

Lösungen: – 13; – 9,5; – 7; – 6; – 3; – 1; – 0,7; – ​ 1__ ​; 0; ​1__ ​; 1; 1; 1; 1,6; 1,7; 2; 3; 5; 5; 6; 6; 6,5; 7; 7 ​3<br />

8 2 ___<br />

16 ​<br />

2<br />

a) Setze das Muster fort.<br />

1. 2. 3. 4. 5. 6. 7. 8.<br />

b) Wie viele Kugeln hast du insgesamt nach dem 8. Schritt verbraucht? 36<br />

n (n + 1)<br />

c) Mit der Formel _______ ​ ​ kannst du die gesamte Anzahl der Kugeln nach dem n-ten Schritt<br />

2<br />

bestimmen. Überprüfe dein Ergebnis von b) und berechne, wie viele Kugeln du bei 24 und 99<br />

Schritten brauchen würdest.<br />

Anzahl der Kugeln bei 24 Schritten: 300 99 Schritten: 4950<br />

3<br />

Welche Terme sind gleichwertig?<br />

In der Reihenfolge der Aufgaben erkennst du das Lösungswort.<br />

ja nein ja nein<br />

a) 3 – 7 x und 7 x – 3 B H b) 4 ∙ ​( x + ​ 1__<br />

2 ​ )​ und 4 x + 2 C E<br />

c) 5 x + 8 ∙ x und 13 x I S d) 0,5 ∙ (3 – 2 x) und 0,5 ∙ 3 – 2 x N E<br />

e) 2 ∙ (– 5 y) und – 10 y L U f) 3 a – 7 a und 4 a S G<br />

g) ​ 1__ ​(x + 6) und 2 + ​1__<br />

3 3 ​x T N h) x + 2 (x – 1) und 3 x – 2 R I<br />

i) a ∙ 2 + 3 ∙ b und 5 ∙ a ∙ b O E k) ​ 1__ x<br />

​x – ​2<br />

2 ___<br />

4 ​ und x S W<br />

Lösungswort: WERTGLEICH<br />

4<br />

Gib zwei verschiedene Terme für den Umfang der Figur an.<br />

a)<br />

2 y<br />

b)<br />

1_<br />

a<br />

3<br />

a<br />

x<br />

x<br />

2<br />

y<br />

a<br />

a<br />

1_ x<br />

2<br />

2 y<br />

1_<br />

x<br />

2<br />

1. Term:<br />

U<br />

<br />

= 12 · __ ​ 1 2 ​ x + 2 x + 4 · 2 y + 2 y U = a + a + a + a + a + a + 2 · 3<br />

2. Term: U = 8 x + 10 y U = 6 a + 6<br />

a<br />

a<br />

49


Gleichungen und Terme<br />

Rechnen mit Termen – Addieren und Subtrahieren<br />

<br />

Fasse so weit wie möglich zusammen.<br />

Die richtigen Ergebnisse führen dich zum Lösungsspruch.<br />

a) 9 x + 3 x = 12 x<br />

b) 3 a – 7 a = – 4 a<br />

c)<br />

1__<br />

2 y + 3__<br />

5 y = 1,1 y<br />

d) 5 r – 2 s + 4 r = 9 r – 2 s<br />

e) 3__<br />

7 __ 3<br />

n – 7 + 7 n = f) 0,8 x + x ∙ 0,8 – 4 ∙ x =<br />

4<br />

4 n – 7<br />

– 2,4 x<br />

g) – 6 x – 9 x – 5 x = – 20 x<br />

h) 3 b – 3 + 3 b – 3 = 6 b – 6<br />

i)<br />

1__<br />

2 a + 1__<br />

3 b – 1__ 1__<br />

4 a + b = 4 a + 1 __ 1<br />

3 b<br />

k) 5 – 2 z + 3 – 3 z + 5 z = 8<br />

l) x – 1 + 3__ – x – __ 1 ___ 7<br />

x__<br />

– 2 x = 4<br />

m)<br />

4 3 + x__<br />

2 – 1__<br />

4 x = 12 x<br />

0 8 – 4 a 1__<br />

4 a + 4__<br />

3 b 1 ___ 7<br />

12 a b 6 b – 6 7 3__<br />

4 n – 7 7 r – s 9 r – 2s<br />

NI MEN RG ER AR ET HA FA IC<br />

– 2,4 x – 20 x 12 x ___ 7<br />

12 x – x – 1__<br />

4<br />

x – 1 1,1 y ___ 3<br />

10 y 3 + 5 z<br />

RT IG NU MEN EH HE LE SEN SE<br />

Lösungsspruch:<br />

NUR GLEICHARTIGE TERME NEHMEN<br />

2<br />

Vereinfache. Das Ergebnis der ersten Aufgabe ist die erste Zahl der zweiten Aufgabe usw.<br />

Wenn du nacheinander die zugehörigen Buchstaben notierst, kannst du das Lösungswort erkennen.<br />

– 7 x + 2 x = – 5 x E x + (6 – x) = 6 F a – 1 + 2 x + 1 = a + 2 x<br />

E 3 – 3 a + 3 a – x = 3 – x N – 2 x + (2 x + 8) = 8<br />

I 2 – 4 a + (a + 1) =<br />

G 8 – 4 x – 6 – 2 = – 4 x F – 5 x – 3 a + 4 x + x = – 3 a E – 4 x + 4 (a + x) =<br />

L – 3 a + 2 – 3 a + 2 a = – 4 a + 2 R 4 a – a – 3 a = 0<br />

E a – 3 x – a + x =<br />

A a + 2 x – x ∙ 5 = a – 3 x N 6 + 2 a – 7 – a = a – 1 G 3 – x + (2 x – 3) =<br />

3 – 3 a<br />

4 a<br />

– 2 x<br />

x<br />

Lösungswort:<br />

F LIEGENFAENGER<br />

<br />

Stelle einen Term auf und vereinfache so weit wie möglich.<br />

a) Subtrahiere von der Summe aus x und dem Doppelten von y das Dreifache von x.<br />

(x + 2 y) – 3 x = – 2 + 2 y<br />

b) Verdreifache die Differenz von a und 4. Addiere dazu die Hälfte dieser Differenz.<br />

3 (a – 4) + __ 1<br />

2 (a – 4) = 3 a – 12 + __ 1<br />

2 a – 2 = 3 __ 1<br />

2 a – 14<br />

c) Subtrahiere von einer natürlichen Zahl n ihren Nachfolger und ihren Vorgänger.<br />

n – (n + 1) – (n – 1) = n – n – 1 – n + 1 = – n<br />

d) Addiere zum Doppelten von x die Differenz aus (– 5) und x.<br />

2 x + (– 5 – x) = x – 5<br />

0


Gleichungen und Terme<br />

Rechnen mit Termen – Multiplizieren und Dividieren<br />

1<br />

a) Multiplikationstabelle b) Multiplikationsmauer<br />

∙ – x 2 a – ​ b__<br />

4 ​<br />

– 16 x y<br />

x – x 2 2 a x – __ ​ 1 4 ​ b x<br />

8 x – 2 y<br />

– 4 4 x – 8 a b<br />

– 4 x – 2 y<br />

​ 1__<br />

2 ​a – ​__<br />

1<br />

2 ​ a x a2 – ​__<br />

1<br />

8 ​ a b<br />

x – 4 0,5 2 y<br />

2<br />

Berechne.<br />

Ein Produkt und ein Quotient haben das gleiche Ergebnis. Finde die Paare.<br />

a) – 5 x ∙ 3 = – 15 x b) 0,3 x ∙ (– 10) = – 3 x c) 2 x ∙ 6y = 12 x y d) ​ 3__ ​ y ∙ (– 5 x) = –3 x y<br />

e) (– 4) ∙ y ∙ (– x) = 4 x y f) 0,2 x ∙ x ∙ 5 = x 2 g) ​ 1__ ​x ∙ 3 x ∙ (– 4) = – 6 x 2 h) x ∙ 2 x ∙ ​2__ x ​= 4 x<br />

– 18 x : 6 = –3 x I 0,4 x : 0,1 = 4 x M 2 x y : ​ 1__ ​= 4 x y F – 12 x y :(– 1) = 12 x y N<br />

x 2<br />

– 15 x<br />

– 5 x 2 :(– 5) = I 30 x :(– 2) = K 15 x y z :(– 5 z) = – 3 x yO ​( – ​ 3__<br />

2<br />

2<br />

5<br />

4 ​x2 )​: ​ 1__<br />

8 ​= – 6 x 2 L<br />

a) b) c) d) e) f) g) h)<br />

K I N O F I L M<br />

3<br />

Wende das Distributivgesetz an.<br />

a) Ausmultiplizieren b) Ausklammern<br />

3 (x + 2) = 3 x + 6 5 ∙ a – 5 ∙ b = 5 (a – b)<br />

​ 2__<br />

4 – ​__<br />

4<br />

​∙ (10 – 2 y) = 5 ​ y<br />

5<br />

8 x – 20 = 4 (2 x – 5)<br />

(– 0,8 + 2 x) ∙ (– 5) = 4 – 10 x<br />

9 y + 9 = 9 (y + 1)<br />

(9 r – 3 s ): 3 = 3 r – s<br />

12 – 4 x + 6 y = 2 (6 – 2 x + 3 y)<br />

c) Multipliziere, ordne und fasse soweit wie möglich zusammen.<br />

3 (x + 5) + (3 – x) ∙ 2 = 3 x + 15 + 6 – 2 x = x + 21<br />

​ x__<br />

2 ​∙ ( ​ – ​ 4__<br />

5 ​ )​+ 0,6 ∙ (x + 10) = <br />

– ​__<br />

2 ​ x + 0,6 x + 6 = 0,2 x + 6<br />

5<br />

2 b + 3 ∙ (a – b) – 5 b + (b – a) ∙ 2 = 2 b + 3 a – 3 b – 5 b + 2 b – 2 a = a – 4 b<br />

4<br />

Löse die Gleichungen.<br />

Tipp: Vereinfache die Terme zunächst so weit wie möglich und benutze dann Umformungen.<br />

a) 3 (x + 7) = 7 x – 4 x + ​ 1__ ​x b) 12 x + (9 x – 6) : 3 = 2 (x – 1)<br />

4<br />

c) 2 (x + 3) + 5 x ∙ (– 4) = 2 x + (x – 6) ∙ 4 d) (2 x – 8) : 2 – 2 x = ​( – ​ 1__<br />

2 ​ )​∙ (10 + 2 x) + 2 x<br />

e) (2 x + 4) ∙ ​ 1__ ​= 2 x– 6<br />

2<br />

f) – 3 x – 4 x – 5 = 3 x + 5 (x + 1)<br />

g) ​ 1__ ​x + ​1__ ​x + ​1__ ​x – 2 = 11<br />

2 3 4<br />

h) 2 x ∙ (– 3) + 3 x ∙ (– 2) = 12<br />

Lösungen: – 1; – ​ 2__ ​; 0; ​1__ ​; 1,25; 8; 12; 84<br />

3 2<br />

51


Gleichungen und Terme<br />

Rechnen mit Termen – Vermischtes<br />

<br />

Ein <strong>Gymnasium</strong> veranstaltet in jedem Jahr mit SPRUNCY (SPonsored RUnning and CYcling) einen<br />

Sponsorenlauf. Kathi bekommt von ihrem Vater 6 € und für jeden gelaufenen Kilometer 2 €. Ihre<br />

Freundin Svea verdient mit jedem gelaufenen Kilometer 3,50 €.<br />

a) Stelle einen Term auf, der das erlaufene Geld der beiden beschreibt.<br />

Kathi:<br />

6 + 2 · x Svea: 3,5 · x<br />

b) Nach wie vielen Kilometern haben beide gleich viel erlaufen?<br />

6 + 2 · x = 3,5 · x x = 4<br />

c) Kathi schafft 6 km, Svea sogar 8 km. Wie viel Geld erlaufen die beiden zusammen?<br />

6 + 2 · 6 + 3,5 · 8 = 46<br />

2<br />

Finde heraus, welche Aufgaben richtig und welche Aufgaben falsch umgeformt wurden.<br />

Bei den richtigen wähle den grünen Buchstaben, bei den falschen den schwarzen.<br />

a) 3 + 1,5 x = 4,5 x V Z b) 15 ∙ ( y – 3__<br />

5 ) = 15 y – 9 T E<br />

c) x__<br />

2 + x ∙ 1__<br />

4 – 2 = 1 1__<br />

2 x – 1 3__<br />

4<br />

R E d) 4 n + 2 (n + 2) = 6 n + 4 S O<br />

e) 5 a – 10 b = 5 (a – 2 b) E A f) 7 a b – b = 7 a A G<br />

g) 2 x + (x + 2) + (x – 2 ) = 4 x N S h) 0,6 ∙ (– 0,5 x) ∙ (– 10) = 3 x E D<br />

i) 8 c d – 4 c ∙ 2d – d = d K H k) (6 – 6 a) : (– 6 ) = a – 1 C N<br />

l) 3__<br />

4 b – 3__<br />

4 a + 1__<br />

2 a ∙ 3__<br />

2 – 1__<br />

2 b = 1__ b E U m) 4 x ∙ 5 y + 4 y ∙ 5 z = 180 x y z L R<br />

4<br />

Lösungswort:<br />

RECHENGESETZ<br />

<br />

Stelle eine Gleichung auf, vereinfache sie so weit wie möglich und löse mithilfe von Umformungen.<br />

Überprüfe dein Ergebnis.<br />

a) Multipliziert man die um drei vergrößerte<br />

Zahl mit 8, erhält man dasselbe, als wenn<br />

man die um 8 verminderte Zahl mit 3 multipliziert.<br />

(x + 3) · 8 = (x – 8) · 3<br />

b) Addiert man zum Doppelten einer Zahl 6<br />

und dividiert das Ergebnis durch (– 4), erhält<br />

man 1 mehr als die Hälfte der Zahl.<br />

(2x + 6) :(– 4) = __ 1<br />

2 x + 1<br />

8 x + 24 = 3 x – 24<br />

1__<br />

2 x – __ 3<br />

2 = __ 1<br />

2 x + 1<br />

48 = – 5 x<br />

– __ 3<br />

2 = x + 1<br />

– 9,6 = x<br />

– __ 5<br />

2 = x<br />

Probe:<br />

– 6,6 · 8 = – 17,6 · 3<br />

– 52,8 = – 52,8<br />

<br />

(– 5 + 6) :(– 4) = – __ 5<br />

4 + 1<br />

– __ 1<br />

4 = – __ 1 <br />

4<br />

2


Geometrische Konstruktionen an Dreiecken<br />

Kongruenzsätze SSS, SWS, WSW<br />

1<br />

Konstruiere ein Dreieck ABC aus den gegebenen Größen. In welchem Fall geht es nicht?<br />

A<br />

A<br />

a)<br />

b)<br />

C<br />

γ<br />

<br />

C<br />

B<br />

B<br />

c)<br />

C<br />

<br />

B<br />

a) a = 3 cm; b = 5 cm;<br />

c = 4 cm<br />

b) a = 4 cm; = 25°;<br />

γ = 100°<br />

c) a = 3 cm; = 50°;<br />

b = 2 cm<br />

In welchem Fall ergibt sich<br />

kein Dreieck?<br />

c)<br />

In welchem Fall entsteht<br />

ein rechtwinkliges<br />

Dreieck?<br />

a)<br />

2<br />

Konstruiere ein Dreieck ABC aus den gegebenen Größen. Miss die drei Innenwinkel.<br />

a)<br />

C<br />

b)<br />

C<br />

γ<br />

a) a = 4 cm; b = 2,5 cm;<br />

c = 3,5 cm<br />

α = 82° = 38°<br />

γ =<br />

60°<br />

A<br />

B<br />

A<br />

B<br />

b) a = 3 cm; b = 2 cm;<br />

γ = 60°<br />

α = 79° = 41°<br />

c = 2,6 cm<br />

3<br />

Konstruiere ein Dreieck ABC aus den gegebenen Größen. Gib die Koordinaten von C an.<br />

5<br />

4<br />

y<br />

C<br />

C a) A (0 | 2); B (3 | 1)<br />

γ<br />

B α = 70°; b = 3,2 cm<br />

C (j 2 | j) 4,5<br />

3<br />

2<br />

A<br />

α<br />

α<br />

A<br />

b) A (6 | 2); B (10 | 5)<br />

α = 45°; γ = 90°<br />

C (j 6,5 | j) 5,5<br />

1<br />

B<br />

x<br />

1 2 3 4 5 6 7 8 9<br />

10<br />

53


Geometrische Konstruktionen an Dreiecken<br />

Kongruenzsatz SsW<br />

<br />

Konstruiere ein Dreieck ABC aus den gegebenen Größen. In welchem Fall geht es nicht?<br />

A<br />

A<br />

a) c = 4 cm; a = 2 cm;<br />

a)<br />

C<br />

α = 55°<br />

b) c = 4 cm; a = 3,5 cm;<br />

c)<br />

α = 55°<br />

c) c = 4 cm; a = 5 cm;<br />

α = 55°<br />

C 2<br />

B<br />

In welchem Fall ergibt sich<br />

kein Dreieck?<br />

b)<br />

a)<br />

In welchem Fall ist die<br />

C 1 Konstruk tion nicht ein-<br />

A<br />

B deutig?<br />

B<br />

b)<br />

2<br />

Konstruiere ein Dreieck ABC aus den gegebenen Größen. Miss die fehlenden Größen.<br />

a)<br />

C<br />

a) a = 3 cm; b = 4 cm;<br />

= 30°<br />

A<br />

B<br />

C<br />

α = 22° ; γ = 128° ;<br />

c = 6,3 cm<br />

b) b = 5 cm; c = 4 cm;<br />

b)<br />

A<br />

1<br />

2<br />

B 1<br />

B 2<br />

γ = 50°<br />

1 = 73° ; α 1 = 57° ;<br />

a 1 = 4,4 cm<br />

2 = 107° ; α 2 = 23° ;<br />

a 2 = 2,1 cm<br />

<br />

Konstruiere ein Dreieck ABC aus den gegebenen Größen. Gib die Koordinaten von C an.<br />

C<br />

C 1<br />

a) A (0 | 1,5); B (3 | 0,5)<br />

b = 4 cm; = 59°<br />

C (j 2 | j) 5<br />

b) A (5 | 2); B (10 | 2)<br />

b = 4,1 cm; = 45°<br />

C 2<br />

6 6 9 3<br />

C 1 (j | j); C 2 (j | j)<br />

A<br />

A<br />

B<br />

B<br />

4


Geometrische Konstruktionen an Dreiecken<br />

Konstruierbarkeit von Dreiecken<br />

1<br />

Welche Dreiecke ABC sind nicht konstruierbar? Begründe.<br />

a) a = 4 cm, b = 8 cm, c = 3 cm j nicht konstruierbar, a + c < b<br />

b) α = 67°, = 95°, γ = 23° j nicht konstruierbar, Winkelsumme > 180°<br />

c) a = 5 cm, b = 6 cm, γ = 100° j nicht konstruierbar, SWS<br />

d) α = 85°, a = 3 cm, c = 6 cm j nicht konstruierbar, a schneidet nicht Schenkel von α<br />

e) α = 15°, = 105°, b = 3 cm j nicht konstruierbar, WSW <br />

2<br />

Das Dreieck ABC soll konsturiert werden. Zwischen welchen Werten liegt die Länge der Seite b?<br />

a) a = 5 cm, c = 7 cm<br />

jj 2 cm < b < jj 12 cm<br />

b) a = 2,5 cm; c = 6,2 cm<br />

jj 3,7 cm < b < jj 8,7 cm<br />

b a<br />

b<br />

b<br />

c<br />

a<br />

a<br />

3<br />

Eine dreieckige Glasscheibe in einem<br />

gotischen Fenster soll ersetzt werden. Dazu<br />

werden vom Glaser einige Größen gemessen.<br />

In welchen Fällen wurde so gemessen, dass<br />

die Glasscheibe eindeutig passend hergestellt<br />

werden kann? Begründe.<br />

b = 5 cm<br />

a = 4 cm<br />

c = 8 cm<br />

a) a = 4 cm, b = 5 cm j eindeutig konstruierbar, <br />

b) b = 5 cm; c = 8 cm; γ = 105° j eindeutig konstruierbar, SsW <br />

c) α = 24°, = 31°, a = 4 cm j eindeutig konstruierbar, WSW <br />

d) a = 4 cm, b = 5 cm; α = 24° j eindeutig konstruierbar, Vor. SsW nicht erfüllt<br />

4<br />

Entscheide, welches Dreieck eindeutig konstruierbar ist. Gib den entsprechenden Kongruenzsatz an.<br />

Begründe, wenn ein Dreieck nicht eindeutig konstruierbar ist.<br />

Dreieck eindeutig konstruierbar Begründung<br />

a) a = 6,5 cm, c = 8,2 cm, α = 70° j Ja j Nein Vor. SsW nicht erfüllt<br />

b) c = 8,2 cm, α = 70°; γ = 55° j Ja j Nein WSW<br />

c) a = 5,6 cm, α = 115°, γ = 68° j Ja j Nein WSW<br />

d) a = 5,6 cm, b = 6,7 cm, γ = 55° j Ja j Nein SWS<br />

e) α = 70°, = 60°, γ = 50° j Ja j Nein<br />

Es entstehen ähnliche Dreiecke.<br />

55


Geometrische Konstruktionen an Dreiecken<br />

Höhe, Seiten- und Winkelhalbierende<br />

<br />

Konstruiere ein Dreieck ABC aus den gegebenen Größen.<br />

A<br />

a)<br />

C<br />

B 2 b)<br />

B 1<br />

a) a = 4 cm; b = 7 cm;<br />

h b = 3 cm<br />

b) a = 4 cm; b = 3 cm;<br />

C w = 5 cm<br />

c) a = 4,8 cm; b = 3 cm;<br />

s<br />

A<br />

a = 3,2 cm<br />

In welchem Fall ergibt sich kein<br />

Dreieck?<br />

In welchem Fall ist die Kon struktion<br />

nicht eindeutig?<br />

c)<br />

a)<br />

In welchem Fall ergibt sich ein<br />

B gleichschenkliges Dreieck?<br />

c)<br />

2<br />

Konstruiere ein Dreieck ABC aus den gegebenen Größen. Miss die fehlenden Größen.<br />

A<br />

b)<br />

a)<br />

A<br />

C<br />

a) w<br />

C<br />

= 3 cm; = 120°;<br />

γ<br />

γ = 25°<br />

a = 7,1 cm; b = 10,7 cm;<br />

c = 5,2 cm<br />

h b<br />

b) α = 50°; = 70°;<br />

B<br />

h b = 3,4 cm<br />

a = 3,9 cm ; b = 4,8 cm;<br />

w <br />

c = 4,4 cm<br />

B<br />

<br />

Konstruiere ein Dreieck ABC aus den gegebenen Größen. Miss die fehlenden Größen.<br />

C<br />

C<br />

a) w = 3,6 cm; h a = 5,3 cm;<br />

= 74°<br />

a = 3,8 cm ; b = 5,8 cm;<br />

c = 5,5 cm<br />

w <br />

α = 39° ; γ = 67°<br />

A<br />

h a<br />

B<br />

b) b = 5,2 cm; c = 3,4 cm;<br />

s b = 3,8 cm<br />

a = 5,6 cm ; α = 77° ;<br />

b)<br />

s b<br />

= 66° ; γ = 37°<br />

A<br />

B<br />

6


Geometrische Konstruktionen an Dreiecken<br />

Anwendungen<br />

1<br />

In welchem Punkt liegt der<br />

Schatz? Du findest ihn, indem<br />

du die beiden Seile spannst.<br />

N<br />

L<br />

M<br />

F H<br />

O<br />

K<br />

G<br />

I<br />

P<br />

Der Schatz liegt im Punkt:<br />

K<br />

E<br />

A<br />

25 m<br />

30 m<br />

D<br />

C<br />

B<br />

35 m<br />

2<br />

Die Cheopspyramide ist<br />

140 m hoch. Unter welchem<br />

Höhenwinkel siehst du sie aus<br />

einer Entfernung von 500 m?<br />

Der Höhenwinkel beträgt 16° .<br />

Das sind<br />

doch nur 8°?<br />

3<br />

4<br />

Jenny und Lennart bestimmen<br />

die Höhe eines Turmes<br />

mit einem Winkelmessgerät<br />

(Theodolit), das auf einem<br />

1,50 m hohen Stativ steht.<br />

Lennart ermittelt 50 m als<br />

Entfernung des Turmes zum<br />

Stativ und Jenny den Winkel<br />

α = 27°.<br />

Die Breite eines Flusses wird<br />

bestimmt, indem von den<br />

Endpunkten einer 60 m langen<br />

Strecke ein markanter Punkt<br />

auf der gegenüberliegenden<br />

Seite angepeilt wird.<br />

Skizze<br />

Wie hoch ist der Turm? Zeichne. Beachte die Stativhöhe.<br />

α<br />

Der Turm ist 27 m hoch.<br />

30° 80°<br />

30° 80°<br />

Der Fluss ist<br />

32<br />

m breit.<br />

60 m<br />

57


Geometrische Konstruktionen an Dreiecken<br />

Raumvorstellung<br />

<br />

Tim, Jule, Michel und Josie haben drei gleiche<br />

Schachteln zusammengeklebt. Jedes Kind sieht<br />

das „Bauwerk“ von einer anderen Seite.<br />

Welches Bild sieht welches Kind?<br />

a) b) c) d)<br />

Michel Josie Jule Tim<br />

2<br />

Die vier Kinder aus Aufgabe 1 haben nun zwei Schachteln zusammengeklebt.<br />

Wie sehen die Kinder das „Bauwerk“?<br />

„Bauwerk“ Michel Josie Tim Jule<br />

<br />

Ordne den Würfelkörpern A bis F ohne Veränderung der Lage die Ansicht von oben zu.<br />

Schreibe die Nummer der Ansicht in den betreffenden Würfelkörper.<br />

Würfelkörper<br />

1<br />

2<br />

1 5 3 6<br />

A<br />

B<br />

C<br />

D<br />

E<br />

F<br />

Ansichten von oben<br />

1 2 3 4 5 6<br />

Für welche Ansicht ist kein Würfelkörper dargestellt?<br />

4<br />

8


Wahrscheinlichkeitsrechnung<br />

Voraussagen mit relativen Häufigkeiten<br />

1<br />

Linus hat seinen Holzwürfel gezinkt. Er hat ihn aufgebohrt, ein Bleistück eingesetzt und dann wieder<br />

unauffällig verschlossen. Er hat mit seinem Würfel anschließend 300-mal gewürfelt:<br />

Augenzahl 1 2 3 4 5 6<br />

Häufigkeit 14 29 31 28 32 166<br />

a) Schätze die Wahrscheinlichkeiten für die einzelnen Ergebnisse ab.<br />

​___<br />

1<br />

20 ​ ___ ​1 10 ​ ​1 ___<br />

10 ​ ___ ​1 ​1 ___<br />

10 ​ 10 ​<br />

___<br />

20 ​11 ​<br />

P (1) ≈<br />

jjj P (2) ≈ jjj P (3) ≈ jjj P (4) ≈ jjj P (5) ≈ jjj P (6) ≈ jjj<br />

b) Bestimme die folgenden Wahrscheinlichkeiten für einen Wurf.<br />

__<br />

P (ungerade) ≈ jj ​ 1 4 ​ P (nicht 6) ≈ jj ​9 ___<br />

20 ​ ___<br />

P (größer als 2) ≈ jj 20 ​17 ​<br />

c) Beim nächsten Mensch-Ärgere-Dich-Nicht-<br />

Spiel muss Linus 100-mal würfeln. Schätze,<br />

wie viele 1en, 2en, … er würfelt.<br />

Augenzahl 1 2 3 4 5 6<br />

Schätzwert 5 10 10 10 10 55<br />

2<br />

Verschiedene Sprachen unterscheiden sich auch dadurch, dass die Buchstaben in unterschiedlicher<br />

Häufigkeit vorkommen.<br />

a) Markiere in beiden Texten alle e rot, alle n blau, alle o grün und alle i gelb. Fülle dann die<br />

Tabellen unter den Texten aus. Runde die relativen Häufigkeiten auf ganze %.<br />

„Mensch, Eva hat schon dreimal hintereinander eine<br />

sechs gewürfelt. Eva kann gut würfeln. Aber beim<br />

nächsten Mal wird sie sicher keine sechs bekommen.“<br />

„Soll ich wirklich mit meinem neuen Freund um die<br />

Wette auf den Basketballkorb werfen? Ich weiß doch<br />

gar nicht, wie gut er trifft. Vielleicht lasse ich besser<br />

die Finger davon.“<br />

So oder ähnlich könnte es lauten, wenn man nicht<br />

genau weiß, wie etwas ausgeht, wenn also der<br />

Zufall ins Spiel kommt. Wie kann man Gewinnchancen<br />

beurteilen? Was versteht man eigentlich unter<br />

Wahrscheinlichkeit, mit der ein bestimmtes Ergebnis<br />

eintritt, und wie kann man diese in konkreten Fällen<br />

ermitteln? Kann man den Zufall vielleicht „berechnen“?<br />

(ca. 550 Buchstaben).<br />

The name of Big Bend National Park comes from<br />

the sharp bend of the Rio Grande River that is part<br />

of the border between the US and Mexico. This<br />

huge park is full of contrasts. Visitors can travel from<br />

the Rio Grande with its spectacular canyons and<br />

<strong>jun</strong>gle-like flood plain up through the Chihuahuan<br />

Desert, which forms the majority of the park, to<br />

the Chisos Mountains with their cool forests. Its<br />

differences in elevation and temperature makes<br />

Big Bend an ideal year-round park. The dessert<br />

areas are very challenging in the summer, with<br />

temperatures going up to 107,6 °F. The basing and<br />

higher Chisos offer backpacking and day hiking on a<br />

number of trails, wildlife watching, camping, hotels,<br />

restaurants and ranger programs throughout the<br />

summer … (ca. 600 Buchstaben).<br />

e n o i<br />

absolute Häufigkeit ca. 90 ca. 60 12 ca. 50<br />

e n o i<br />

absolute Häufigkeit ca. 60 ca. 40 ca. 40 ca. 40<br />

relative Häufigkeit ≈ 16 %≈ 11 % ≈ 2 % ≈ 9 % relative Häufigkeit ≈ 10 %≈ 7 % ≈ 7 % ≈ 7 %<br />

b) Ein deutsches Buch hat 400 ziemlich gleichmäßig beschriebene Seiten. Maria hat auf den ersten<br />

5 Seiten ca. 1200 Buchstaben gezählt. Schätze, wie viele „e“ in diesem Buch stehen.<br />

1200 : 5 · 400 · 0,16 = 15 360 Es sind ca. 15 000 „e“ in dem Buch.<br />

c) Wie viele Seiten müsste ein englisches Buch mit gleicher Schriftgröße und gleicher Seitengröße<br />

ungefähr haben, um ungefähr auf die gleiche „e“ Anzahl zu kommen?<br />

Das Buch müsste ca. 640 Seiten haben.<br />

d) Mit welchen der oben genannten Buchstaben kann man Deutsch und Englisch gut unterscheiden,<br />

bei welchen sollte man vorsichtig sein?<br />

Gute Unterscheidung: „o“ Schlechte Unterscheidung: „i“<br />

Die Buchstaben „e“ und „n“ liegen dazwischen.<br />

59


Wahrscheinlichkeitsrechnung<br />

Theoretische Wahrscheinlichkeiten 1<br />

<br />

Färbe die Glücksräder entsprechend der angegebenen Wahrscheinlichkeiten und ergänze die fehlenden<br />

Wahrscheinlichkeiten.<br />

a)<br />

b)<br />

c)<br />

rot<br />

rot<br />

rot<br />

gelb<br />

gelb blau<br />

blau<br />

gelb<br />

blau<br />

P (rot) = 1__<br />

6<br />

P (gelb) = 0,5<br />

P (blau) = 25 %<br />

___ 1<br />

P (weiß) =<br />

jj 12<br />

P (rot) =<br />

1__<br />

3<br />

P (blau) = 1__<br />

8<br />

P (gelb) = 12,5 %<br />

___ 5<br />

P (weiß) =<br />

jj 12<br />

P (blau) = 2 ∙ P (rot)<br />

P (gelb) = P (blau) + P (rot)<br />

P (weiß) = 0<br />

d)<br />

blau<br />

e)<br />

blau<br />

f)<br />

blau<br />

grün<br />

rot<br />

rot<br />

rot<br />

gelb<br />

1__<br />

P (grün) =<br />

jj 2<br />

P (rot) + P (blau) = 1 – P (grün)<br />

P (rot) = 1__<br />

8<br />

3__<br />

8<br />

0<br />

P (blau) =<br />

jj P (weiß) = jj<br />

1__<br />

P (grün) =<br />

jj 3<br />

P (rot) + P (blau) = 1 – P (grün)<br />

P (rot) = P (blau)<br />

P (weiß) =<br />

jj 0<br />

P (grün) = ___ 1<br />

16<br />

P (rot) = 2 ∙ P (grün)<br />

P (gelb) = 2 ∙ (rot)<br />

P (blau) = 2 ∙ P (gelb)<br />

___ 1<br />

P (weiß) =<br />

jj 16<br />

2<br />

In einer Kiste befinden sich 24 Kugeln (eine weiße, drei blaue, vier grüne, sechs rote, zehn orange).<br />

a) Bestimme die Wahrscheinlichkeiten der folgenden Ereignisse:<br />

E 1 : Eine grüne Kugel ziehen<br />

E 2 : Eine rote oder orange Kugel ziehen<br />

E 3 : Weder eine weiße noch eine blaue Kugel ziehen<br />

E 4 : Eine rote Kugel ziehen<br />

E 5 : Eine gelbe Kugel ziehen<br />

1__<br />

P (E 1 ) =<br />

jj 6<br />

2__<br />

P (E 2 ) =<br />

jj 3<br />

5__<br />

P (E 3 ) =<br />

jj 6<br />

1__<br />

P (E 4 ) =<br />

jj 4<br />

P (E 5 ) =<br />

jj<br />

0<br />

b) Moritz möchte auf einem Straßenfest eine Tombola veranstalten. Er hat den folgenden<br />

Gewinnplan erarbeitet:<br />

Farbe weiß blau grün rot orange<br />

Gewinn 11 € 3 € 4 € 2 € Niete<br />

Max möchte an Moritz’ Stand 96-mal ziehen. Mit welchem Gewinn kann er ungefähr rechnen?<br />

___ 1<br />

24 · 96 · 11 € + __ 1<br />

8 · 96 · 3 € + __ 1<br />

6 · 96 · 4 € + __ 1 · 96 · 2 € = 192 €<br />

4<br />

Wie viel muss Moritz pro Spiel mindestens als Einsatz verlangen, um keinen Verlust zu machen?<br />

192 € : 96 = 2 €<br />

60


Wahrscheinlichkeitsrechnung<br />

Theoretische Wahrscheinlichkeiten 2<br />

1<br />

Karola und Manfred haben für sich die „Mensch-Ärgere-Dich-Nicht“-Regeln verändert. Sie würfeln<br />

mit einem „normalen“ Würfel und einem Würfel, der zwei rote, zwei blaue und zwei gelbe Seiten<br />

hat. Bei rot verfällt die Augenzahl des normalen Würfels, bei gelb wird die normale Augenzahl<br />

gesetzt, bei blau aber verdoppelt sich die Augenzahl.<br />

a) Liste alle möglichen Ergebnisse in der Tabelle auf.<br />

1 2 3 4 5 6<br />

rot 1, R 2, R 3, R 4, R 5, R 6, R<br />

blau 1, B 2, B 3, B 4, B 5, B 6, B<br />

gelb 1, G 2, G 3, G 4, G 5, G 6, G<br />

b) Bestimme die Wahrscheinlichkeiten und fülle die Tabelle aus.<br />

Ereignis Wahrscheinlichkeit Ereignis Wahrscheinlichkeit<br />

Augenzahl 1 ​___<br />

1<br />

18 ​ gerade Augenzahl ​1 __<br />

2 ​<br />

Augenzahl 6 ​___<br />

2<br />

18 ​ Augenzahl 0 ​1 __<br />

3 ​<br />

Augenzahl 8 ​___<br />

1 ​<br />

18<br />

Augenzahl kleiner 7 ___ ​15<br />

18 ​ = __ ​5 6 ​<br />

ungerade Augenzahl ​___<br />

3<br />

18 ​ = __ ​1 6 ​ Augenzahl größer 8 ​1 __<br />

9 ​<br />

c) Beim normalen Spiel darf man bei einer 6 einen neuen Spielstein ins Spiel bringen. Karola<br />

und Manfred stellen fest, dass es nun zu schwer ist, einen Stein ins Spiel zu bringen. Sie lassen<br />

zusätzlich die 12 zu. In welchem Spiel sind die Einsetzchancen größer?<br />

„normal“: ​__<br />

1 ​ „neu“: ___ ​2<br />

6 18 ​ + ___ ​1 18 ​ = ___ ​3 18 ​ = __ ​1 ​ Die Einsetzchancen sind gleich.<br />

6<br />

2<br />

Ein besonderer Würfel hat 20 gleiche Seiten. Diese sollen mit den Farben Rot, Blau, Grün und Gelb<br />

bemalt werden. Dabei sollen sich die folgenden Wahrscheinlichkeiten ergeben:<br />

P (rot) = 0,2<br />

P (blau) = ​ 1__<br />

Ergänze:<br />

Farbe Rot Blau Grün Gelb<br />

Anzahl der Seiten 4 5 8 3<br />

0,15<br />

4 ​ P (grün) = 40 % P (gelb) = jjj .<br />

3<br />

4<br />

In einer großen Kiste befinden sich sehr viele weiße Murmeln. Nikolas hat keine Lust, die Murmeln<br />

zu zählen. Er markiert 50 Murmeln mit einem roten Punkt und wirft sie zurück in die Kiste.<br />

Er mischt gut und zieht 50 Murmeln. Von diesen haben 5 einen roten Punkt.<br />

a) Wie groß ist ungefähr die Wahrscheinlichkeit, eine Murmel mit rotem Punkt zu ziehen?<br />

<br />

​___<br />

5<br />

50 ​ = ___ ​1 10 ​<br />

b) Wie viele Murmeln sind ungefähr in der Kiste?<br />

Es sind ca. 500 Murmeln in der Kiste.<br />

In einem Eimer befinden sich 2000 Lose. Laura zieht 20 Lose. Sie hat 10 Nieten, 5 Trostpreise,<br />

3 Gewinne im Wert von 2 € und 2 Gewinne im Wert von 5 €.<br />

Schätze, wie viele Lose von der entsprechenden Sorte ungefähr im Loseimer sind.<br />

Art des Gewinns Niete Trostpreis Gewinn 2 € Gewinn 5 €<br />

Zahl der Lose im Eimer ca. 1000 ca. 500 ca. 300 ca. 200<br />

61


Wahrscheinlichkeitsrechnung<br />

Zufallsversuche und Baumdiagramme 1<br />

<br />

a) Meta hat die vier Damen eines Kartenspieles<br />

herausgesucht und gemischt. Nun deckt sie<br />

die vier Karten mit den vier Farben Kreuz (Kr),<br />

Pik (P), Herz (H) und Karo (K) nacheinander<br />

auf.<br />

Vervollständige das Baumdiagramm. Schreibe<br />

die Wahrscheinlichkeiten an die Pfade und<br />

gib die Pfadwahrscheinlichkeiten an.<br />

b) Gesa und Enes spielen ein Würfelspiel.<br />

Dazu benutzen sie zwei<br />

Tetraeder, die mit den Zahlen<br />

1 bis 4 beschriftet wurden.<br />

Vervollständige das Baumdia gramm. Schreibe<br />

insbesondere die Wahrscheinlich keiten an<br />

die Pfade und gib hinter den Pfaden die Pfadwahrscheinlichkeit<br />

an. Nutze die Hilfslinie.<br />

1__<br />

4<br />

1__<br />

4<br />

1__<br />

4<br />

1__<br />

4<br />

Kr<br />

P<br />

H<br />

K<br />

1__<br />

3<br />

1__<br />

3<br />

1__<br />

3<br />

1__<br />

3<br />

1__<br />

3<br />

1__<br />

3<br />

1__<br />

3<br />

1__<br />

3<br />

1__<br />

3<br />

1__<br />

3<br />

1__<br />

3<br />

1__<br />

3<br />

P<br />

H<br />

K<br />

Kr<br />

H<br />

K<br />

Kr<br />

P<br />

K<br />

Kr<br />

P<br />

H<br />

1__<br />

2 1__<br />

2<br />

1__<br />

2<br />

1__<br />

2<br />

1__<br />

2<br />

1__<br />

2<br />

1__<br />

2<br />

1__<br />

2<br />

1__<br />

2<br />

1__<br />

2<br />

1__<br />

2<br />

1__<br />

2<br />

1__<br />

2<br />

1__<br />

2<br />

1__<br />

2<br />

1__<br />

2<br />

1__<br />

2<br />

1__<br />

2<br />

1__<br />

2<br />

1__<br />

2 1__<br />

2<br />

1__<br />

2<br />

1__<br />

2<br />

1__<br />

2<br />

H<br />

K<br />

P<br />

K<br />

P<br />

H<br />

H<br />

K<br />

Kr<br />

K<br />

Kr<br />

H<br />

P<br />

K<br />

Kr<br />

K<br />

Kr<br />

P<br />

P<br />

H<br />

Kr<br />

H<br />

Kr<br />

P<br />

1<br />

1<br />

1<br />

1<br />

1<br />

1<br />

1<br />

1<br />

1<br />

1<br />

1<br />

1<br />

1<br />

1<br />

1<br />

1<br />

1<br />

1<br />

1<br />

1<br />

1<br />

1<br />

1<br />

1<br />

K<br />

H<br />

K<br />

P<br />

H<br />

P<br />

K<br />

H<br />

K<br />

Kr<br />

H<br />

Kr<br />

K<br />

P<br />

K<br />

Kr<br />

P<br />

Kr<br />

H<br />

P<br />

H<br />

Kr<br />

P<br />

Kr<br />

___ 1<br />

24<br />

___ 1<br />

24<br />

___ 1<br />

24<br />

___ 1<br />

24<br />

___ 1<br />

24<br />

___ 1<br />

24<br />

___ 1<br />

24<br />

___ 1<br />

24<br />

___ 1<br />

24<br />

___ 1<br />

24<br />

___ 1<br />

24<br />

___ 1<br />

24<br />

___ 1<br />

24<br />

___ 1<br />

24<br />

___ 1<br />

24<br />

___ 1<br />

24<br />

___ 1<br />

24<br />

___ 1<br />

24<br />

___ 1<br />

24<br />

___ 1<br />

24<br />

___ 1<br />

24<br />

___ 1<br />

24<br />

___ 1<br />

24<br />

___ 1<br />

24<br />

1__<br />

4<br />

1__<br />

4<br />

1__<br />

4<br />

1__<br />

4<br />

1<br />

2<br />

3<br />

4<br />

1__<br />

4<br />

1__<br />

4<br />

1__<br />

4<br />

1__<br />

4<br />

1__<br />

4<br />

1__<br />

4<br />

1__<br />

4<br />

1__<br />

4<br />

1__<br />

4<br />

1__<br />

4<br />

1__<br />

4<br />

1__<br />

4<br />

1__<br />

4<br />

1__<br />

4<br />

1__<br />

4<br />

1__<br />

4<br />

1<br />

2<br />

3<br />

4<br />

1<br />

2<br />

3<br />

4<br />

1<br />

2<br />

3<br />

4<br />

1<br />

2<br />

3<br />

4<br />

___ 1<br />

16<br />

___ 1<br />

16<br />

___ 1<br />

16<br />

___ 1<br />

16<br />

___ 1<br />

16<br />

___ 1<br />

16<br />

___ 1<br />

16<br />

___ 1<br />

16<br />

___ 1<br />

16<br />

___ 1<br />

16<br />

___ 1<br />

16<br />

___ 1<br />

16<br />

___ 1<br />

16<br />

___ 1<br />

16<br />

___ 1<br />

16<br />

___ 1<br />

16<br />

Fülle die Tabelle aus.<br />

Fülle die Tabelle aus.<br />

Ereignis<br />

P (Ereignis)<br />

rote und schwarze Karten ab-<br />

1__<br />

wechselnd<br />

3<br />

keine Übereinstimmung mit der<br />

___<br />

9<br />

Reihenfolge Kr, P, H, K 24 = __ 3 8<br />

genau eine Übereinstimmung<br />

___<br />

8<br />

mit der Reihenfolge Kr, P, H, K 24 = __ 1 3<br />

genau zwei Übereinstimmungen<br />

mit der Reihenfolge Kr, P,<br />

___<br />

6<br />

H, K<br />

24 = __ 1 4<br />

genau drei Übereinstimmungen<br />

mit der Reihenfolge Kr, P, H, K 0<br />

Ereignis<br />

P (Ereignis)<br />

Pasch __<br />

1 4<br />

Augensumme 5 __<br />

1 4<br />

Augensumme größer als 5 __<br />

3 8<br />

Augenprodukt 4 ___<br />

3<br />

16<br />

Augenprodukt ist ungerade __<br />

1 4<br />

Augensumme ist eine Primzahl ___<br />

9<br />

16<br />

62


Wahrscheinlichkeitsrechnung<br />

Zufallsversuche und Baumdiagramme 2<br />

1<br />

a) Ordne die vier Sachverhalte den Baumdiagrammen zu und fülle diese vollständig aus. Schreibe<br />

die Wahrscheinlichkeiten an die Linien. Notiere hinter jedem Pfad die Pfadwahrscheinlichkeit.<br />

Sachverhalt 1:<br />

Im letzten Arbeitsschritt werden Taschenrechner<br />

(TR) von drei Kontrolleuren nacheinander auf Funktionsfähigkeit<br />

getestet. Der erste Kontrolleur schafft<br />

es, 80 % der fehlerhaften Stücke zu finden, der zweite<br />

findet von den übrigen Fehlerhaften noch einmal<br />

60 %, der dritte kommt noch auf 40 %. Ein Kunde<br />

interessiert sich für die fehlerhaften TR, die durch<br />

die Kontrolle kommen.<br />

Sachverhalt 2:<br />

Bei einem Adventure-Spiel hat man in einem Raum<br />

die Wahl zwischen zwei gleichen Türen. Man<br />

kommt jeweils in einen Raum mit drei gleichen<br />

Türen. In dem einen Raum führt eine der drei Türen<br />

in einen Kerker, die anderen führen ins Freie. In dem<br />

anderen Raum ist es genau umgekehrt.<br />

Sachverhalt 3:<br />

Lars fährt mit dem Fahrrad zur Schule. Unterwegs<br />

überquert er zwei Ampeln. Bei der ersten ist 60 s rot,<br />

30 s grün und 10 s gelb. Die zweite ist eine Fahrradampel<br />

mit 40 s rot und 20 s grün.<br />

Sachverhalt 4:<br />

In einer Urne befinden sich 5 rote, 10 blaue und 25<br />

gelbe Kugeln. Es wird zweimal ohne Zurücklegen<br />

gezogen.<br />

Sachverhalt 2 Sachverhalt 3<br />

A<br />

B<br />

​ 1__<br />

3 ​<br />

​<br />

__ 1<br />

​<br />

__ 1 ​ 1__<br />

2 ​<br />

3 ​<br />

6 ​<br />

​<br />

__<br />

K<br />

2 3 ​ R 0,4<br />

​ 1__ ​<br />

R<br />

T 1 F<br />

​ 1__<br />

6<br />

3 ​ ​ 1__<br />

0,6<br />

​<br />

​<br />

__ 1<br />

3<br />

F<br />

​ 2__<br />

6 0,3<br />

​<br />

Gr 0,2<br />

3 R 0,2<br />

​ 1__ ​<br />

​ 1__<br />

2 ​ 3<br />

​ 1__ ​<br />

Gr<br />

​ 1__ ​ F 6<br />

T 2 3 K ​ 1__ ​<br />

​<br />

​ 1__<br />

6 0,1<br />

​ 2__ 1__ ​ Gr 0,1<br />

​ 3 3 R<br />

___ ​ 2<br />

​ ​ 1__ ​<br />

30<br />

Ge<br />

​<br />

3 K 6 ​ 1__ ​ ___<br />

Gr ​ 1<br />

3 30 ​<br />

Sachverhalt 4<br />

Sachverhalt 1<br />

C<br />

___<br />

​<br />

_____<br />

​ 4 R<br />

20<br />

1560 ​<br />

D<br />

39 ​ R<br />

___ ​ 10<br />

39 ​ B<br />

____ ​ 50 ​<br />

___<br />

1560<br />

​ 25<br />

39 ​ G ____ ​ 125 ​<br />

​<br />

___ 5<br />

1560<br />

40 ​<br />

___<br />

___ ​ 5<br />

____ ​ 50<br />

​ 10<br />

​<br />

39 ​ R<br />

40 ​ 1560<br />

B ​ ____<br />

0,6 80 ​<br />

NA 0,048<br />

___<br />

B ​ 9<br />

___<br />

1560<br />

​ 25<br />

G<br />

____ ​ 250<br />

39 ​<br />

39 ​<br />

0,4 NA<br />

​<br />

0,2 NA<br />

0,4 A<br />

1560<br />

​<br />

___ 25<br />

40 ​ 0,6 A<br />

___ ​ 5<br />

R<br />

____ ​ 125<br />

39 ​ ​<br />

0,8 A<br />

___<br />

1560<br />

G ​ 10<br />

B<br />

____ ​ 250 ​<br />

39 ​ 1560<br />

___<br />

​ 24<br />

G ​ ____ 600 ​<br />

A: Aussortiert<br />

39 ​ 1560 NA: Nicht aussortiert<br />

b) Beantworte die Fragen zu den vier Sachverhalten:<br />

Mit welcher Wahrscheinlichkeit kommt ein fehlerhafter TR in den Verkauf?<br />

Ein fehlerhafter TR kommt mit einer Wahrscheinlichkeit von 4,8 % in den Handel.<br />

Mit welcher Wahrscheinlichkeit kommt man als Spieler ins Freie?<br />

<br />

Man kommt mit einer Wahrscheinlichkeit von __ ​ 1 ​ ins Freie.<br />

2<br />

Mit welcher Wahrscheinlichkeit muss Lars mindestens einmal stehen bleiben?<br />

Lars muss mit einer Wahrscheinlichkeit von 0,9 mindestens einmal stehenbleiben.<br />

Mit welcher Wahrscheinlichkeit zieht man zwei verschiedene Kugeln?<br />

Mit einer Wahrscheinlichkeit von _____ ​<br />

<br />

850 ​ zieht man zwei verschiedene Kugeln.<br />

1560<br />

63


Wahrscheinlichkeitsrechnung<br />

Zufallsversuche und Baumdiagramme 3<br />

<br />

Irina und Tobias veranstalten eine Tombola. Es befinden sich jeweils eine rote, fünf gelbe und<br />

vier blaue Kugeln in einer Socke. Irina lässt mit Zurücklegen, Tobias ohne Zurücklegen ziehen.<br />

Spielregeln: Wird eine blaue Kugel gezogen, so ist das Spiel beendet. Bei einer roten oder gelben<br />

Kugel muss der Spieler weiterziehen. Nach der dritten Ziehung ist das Spiel in jedem Fall beendet.<br />

a) Ordne die Baumdiagramme Tobias und Irina zu und fülle sie aus.<br />

Name: Tobias<br />

Name: Irina<br />

_____ 1<br />

___ 1<br />

R 1000<br />

10<br />

5__<br />

___ 5<br />

4__<br />

_____ 5<br />

9<br />

8 ___ 1<br />

R 10 G<br />

R G 4__<br />

___ 4<br />

1000<br />

G<br />

8 36<br />

_____ 4<br />

___ 1 10<br />

4<br />

___ 1<br />

___ 1 B<br />

1000<br />

___<br />

4__<br />

B<br />

10<br />

36<br />

10 ____ 5<br />

B 90<br />

G R<br />

9<br />

5<br />

1000<br />

___ ___ 4 ___ 5 _____ 25<br />

___ 1<br />

4__<br />

10<br />

10<br />

8<br />

___ 1<br />

10 10 G 1000<br />

R G 36<br />

___ 4<br />

_____ 20<br />

R B<br />

1__<br />

4__<br />

___ 1<br />

10 B<br />

___ 4<br />

1000<br />

9<br />

8 B 36 10 1<br />

____ 5<br />

___<br />

___ 1 ___ 1<br />

R 1000<br />

___ 5<br />

1__<br />

10<br />

4__<br />

R 36 10<br />

10<br />

9<br />

8 R<br />

___ 5 ____ 25<br />

G<br />

3__<br />

___ 1<br />

G G G 1__<br />

10<br />

___ 4 10 1000<br />

____ 20<br />

8<br />

12<br />

B<br />

4__<br />

1__ ___ 5<br />

___ 5<br />

10 ___ 5 1000<br />

B<br />

8<br />

9 10<br />

G<br />

10<br />

10<br />

G G 1_<br />

___ 1<br />

4__<br />

8<br />

10<br />

____ 25<br />

___ 4<br />

R<br />

9<br />

___ 4<br />

1000<br />

10<br />

2__<br />

___ 4<br />

1__<br />

B 9<br />

10 B 10 _____ 100<br />

5 B<br />

___ 4<br />

1000<br />

___ 4<br />

10<br />

B 10<br />

4<br />

B ___<br />

10<br />

b) Bestimme mit den Pfadregeln die Wahrscheinlichkeiten der Spielausgänge.<br />

Spiel mit Zurücklegen<br />

Spiel ohne Zurücklegen<br />

P (dreimal rot) 0,001 0<br />

P (zweimal rot, einmal gelb) 0,015 0<br />

P (einmal rot, zweimal gelb) 0,075<br />

___ 1<br />

12 = 0,083<br />

P (dreimal gelb) 0,125<br />

___ 1<br />

12 = 0,083<br />

P (irgendwann blau) 0,784<br />

__ 5<br />

6 = 0,83<br />

c) Gewinnpläne:<br />

Irina Preis im Wert von… Tobias Preis im Wert von…<br />

3 × R 100 € 2 × G, 1 × R 9 €<br />

1 × G, 2 × R 10 € 3 × G ?<br />

1 × R, 2 × G 4 €<br />

3 × G 2 €<br />

Irina verlangt für ein Spiel 1 € Startgeld. Wie viel Gewinn wird sie bei 1000 Spielen voraussichtlich<br />

machen?<br />

1000 ·(0,001 · 100 € + 0,015 · 10 € + 0,075 · 4 € + 0,125 · 2 €) = 800 € Irina wird voraussichtlich ca. 200 € Gewinn<br />

Welchen Gewinn darf Tobias im Falle dreimal Gelb ausschütten, um auf Dauer bei einem Einsatz machen.<br />

von 1 € weder Gewinn noch Verlust zu machen?<br />

1 € – ___ 1 · 9 € = 0,25 €<br />

12<br />

0,25 € : ___ 1<br />

= 3 €<br />

12<br />

Im Falle „dreimal gelb“ darf er bis zu<br />

3 € ausschütten.<br />

64

Hurra! Ihre Datei wurde hochgeladen und ist bereit für die Veröffentlichung.

Erfolgreich gespeichert!

Leider ist etwas schief gelaufen!